Sie sind auf Seite 1von 179
DESIGN OF CONCRETE STRUCTURES BY M.L. GAMBHIR ed References 1321 Tension reinforcement 13.22 Compression reinforcement 1323 Anchorage 1324 Shear reinforcement 13.25 The detailing of support points . 13.26 Curtailment of flexural steel Detailing the Compression Member 133.1 Longitudinal reinforcement 13.32 ‘Transverse reinforcement Detailing the Beam—Columa Joints 134.1 Comer joints 1342 Exterior and interior joints ‘Symbols for Detailing A: Dead Weights of Some Common Building Materials B: Live Loads on Floors and Roofs C: Reinforcement Bars and Reinforcement Mesh D: Weight of Steel in Slabs E: Basic Horizontal Seismic Coefficients for Some Important Indian Towns af Contents"*iy 335 345 347 350 354 357 359 were INTRODUCTION TO REINFORCED CONCRETE 1.1 GENERAL, ‘The popularity fe eener is ete fat a fom the common ingretct,is osle oar ‘te properties oconcreswmet te domes fey parca seaion Te ears ine oe Save ped the way to make ts teal ue of tally alae murs by olcoes ois eetco oe oper woman, soso pours cmt sang porate pean ‘The fied product hardened concrete) has high compesivesrengh bois ens cengthis very low ayy : a stunsons whe ene sees eee engined sel ba fang a compas consrvctoncaledvelyoredconeet recs Tac cnerete wins reacumen is caled pan coneree apc “Tous, cone making sot just a mater of mixing ingeiet to rodce aplastic mas, but goad * cance soe te performance remeron ic ta se tc honed fe {ne past state he coors sbold be workable andes Gs ser tecion and becing. Sere ‘Cement, aggregates, water and admixtures, and the resulting product. For more details the discerning readers ‘can refer to the author's book cattle: Conerete Technology. * tn addition, types and peopertie ofthe rei forcement are described, and a general overview of basic concrete sirscturl systems is ven, 12 CONCRETE-PRODUCING MATERIALS 12.4 Cement (Cements a finely ground material having adhesive and cuhesive properties, which provide a binding medium, {or the discrete ingredients tls produced by burning together na definite proportion, a mixture of nalually SS "Foc dts, ee Selected Reference, 2 Reinforced Concrete Design ‘occuring arillaceoas(contaning alumina) ad calcareous (containing calcium carbonate or lime) materials, {oA partial fusion at about 14S0°C. The product obained on buming, called clinker (he cement pellet product) is cooled and grouid toa required fineness o produce a material known as cemeat. During grinding of einer ‘a‘small percentage (about 3 por cent by weight of clink) of gypsum is added to consol of retard the time of seting of cement in the Feld, L ‘The main oxide composition of ordinary Pontand cements, typically, lime (CaO) 60-65% silica (SiO,) 17-25%, alumina (Al,0,) 3-88, iron oxide (Fe,0,) 0.56%. Differeat types of Pordand cements ae obained ‘by varying the relative proportions ofthese four predominant compounds and by grinding clinker to different ‘degrees of finenes. The two most common types of cements ar: ordinary Portland cement (OPC) and rapid hardening Portland cement. The oxides prescat inthe cement interact wit each otber to forma series of moxe complex products during fusion, namely, wicalcium silicate (abbreviated as C,S), dcalcium silicate (CS), \wicalejum aluminate (C,A) and tetracalciom alumino ferrite (C,AF). The two silicates, namely C,S and CS, ‘hich together constitute about 70 to 80 per cent of cement, control mast ofthe strengb-eiving properties, Hydration: When cements mixed with water, he various compounds of cement begin to react chemically with water. This reaction is called Jdravon, which resulis in rcrystalizaion in te form of iserlocking ‘rysal that pvnce the cement gel that hat high compressive strength when it harden Ticaciom eliate (CS), having faster rte of reaction accompanied by greater evoluion of heat, developseaily strength, whi ‘calcium silicate (C,S) bydcates and hardens slowly and provides mich ofthe ulimate strength. ‘The rate of hydration inereases withthe fineness to which the cement has been ground. In peactce, te fineness is mensured by specific surface of cement particles. The specific surface of ordinary Poland coment ‘ust not be less tian 225,000 mn/g, For ordinary Porland coment praccally al partcies wil pas irugh ‘the 150-pun IS sieve and over 90 per cent wil pass rough the 904m I sieve, During seting and hardening lime is liberated which is approximately 20 per cet by weight of cement. ‘Under unfavourable conditions excessive expansion of fee line occur afer the cementhas tet sachespansion causes cracking, disruption and disinzgration of the concrete mast, and hence may threaten the safety ofthe uvcture. This undesirable expansion is termed asthe unsourdness of cement. Addtion of siliceous materials suchas pozzolana to cement may belp in preventing such @ condition. Setting time: Ceaneat whe’ mixed with water forms a paste which gradually becomes less plastic, and foally a hard mats is obtained. In this process of hydration and seting, a stage is reached when the cement paste is suficiently sti to withstand a prescribed pressure, The time to reach this sages termed setting tine. ‘The time is reckoned from the insian when waters added to the cement. The ime a which the cement paste loses is plasticity and jst withstand the prescribed pressure is termed the initial eting md, Similarly, the ‘ime taken to reach te stage when the paste becomes a ard mass Is known asthe faa sein Lime. Wisesseatial for proper concreting thatthe inidal sting ime be suffcienly long for Gnishing operations, ic. transporting and proper placing i the formwork. Foran ordinary Porand cement, the inital ting tine ‘must be atleast 30 min and Gna seting dine should beat mast 600 min to allow the early se ofthe serve. ‘Compressive strength: The suength tests of cement are usually cried out on cemeat morta cubes having ‘face area of $000 mar In this test cement is gauged with standard sand, in proportion (by weigh) of oe. [art cement and ee parts of standard sand, and with a waler-cement ratio of 040 (by weight). The cubes ‘remade, curedand tested under carefully consoled conditions. Forordinary Portia comet the compression, strength U3 and 7 days of curing shall not be less than 16 N/oun* and 22 Ninn’ especively. Unit weight: The specific gravity of cement generally Wis witha the range 3.11032, For most calculations taken as 3.15. The unit weight of bulk cement depends upon ihe degree of compaction a a Tough Buide may be taken a5 1450 kgm’. A bag of cement weighs about 50 Kg. 1.22 Aggregates ‘The ageregate is used primarily forte purpose of providing bulk vo the concrete, and constitutes 0 vo $0 “Wo finished product, To inerease the destyof te resulig mix, te aggregae i frequent used ia ‘esizes. The most important function of smaller size panicles (ine aggregate) istoassstinprodacing, ‘1d unify inthe mixture. The ine aggregate also assists Ue cement paste to Mold We lager S ” Inuoducion wo Reinforced Concrete 3 articles (coarse aggregate) in suspension. This action promos plasty in the mixture ad prevents the pessible segregation of paste and coerse agaegas pureulary when ls eceseary to rasp e Cone Sonne distance from the mixing plant ote point of placement, Clasifcaion: The cissiication of aggregates usd in concrete probicton is enerly based on thee seologia origin, siz, shape, uit weight es However, bere nly he Casication econ to sz il be ‘casted. According osx We aggregates clasiied ne agree, corse aggrezatall-n AEST, nd siglestze aggregate. Fine ggrepne: isan aggregate suchas natural sand or enh stone snd or crushed gravel sand, which ‘consists of particles, mos of whic ass rough a4.75 mm IS sieve. The sands generally considered wo have ‘lower sizlimit of bout 07 mm, Accor oie te ie agree maybe described a cons, meu, and ine sands. Depending up ie pris ie csiton, 1385 197 kas vided Ge five areas nto Tour grading zones. The grating zoos become progressively fe ram gang none Io grading one 1. Coarse aggregate: Its an aggregate sch as cused soe, rished gravel or uncrusbed gravel most Of hich retained on 475 mn TS sieve. The graded ease aggregate is esrbed by ts nomial le, 1240 tm, 20 mum, 16 mm, 10 mum, and oon, For example, a rode agree of nomial sizeof 10 mun means ‘aggregate, most o whieh paces hough the 10min 1 Seve Aili aggregate: Sometimes combined agregites ae avaiable in nature comprising diferent actions of ie and couse aggeznex, which are Know alin agerepte, Single sie agaregae: An eggreguc comprising particles fling eselally within anarow limit of size Sexton i cll single-sie operate. Forexample 220mm single ze aggregate means an agregate, most of which passes tough 20 a, 1S sieve andthe major portion of wich ie ened ona TO 6 seve. ‘The nominal size tobe usd ina job depends upon the dimensions ofthe concrete member, clear cover torcinforement, andthe distance bxweea enforcing tars The lage the nomial sie, he lows wil be ie ntetcoment aia Loc ie Ssied workabiiy and a higher sgt oBened op toa nominal maximum ‘size of 40.qun, —— “The aggrepnte to be wsedinconcre production should be durable ad chemically inert under contons to whict it willbe exposed. Oe imporatrequireneas concer the shape, te Surface temue, a te {ading ofthe aggregate. Depending upen te particle tape, the apgepae may be casiNed as angular, ‘regular (or party round) nd rownded. Based on trace characteristic, te aggregate Is cssifed ls, smooth, granu, crysaline,boneycombed, porous, nd 30 on. The shape texture and roughness of Aggrepessiglicanl iniuence ch moby Gx. workability of fe cence andthe bond between the segregate andthe mortar phase, The strength of concrete, epoca the flzuralsrength dopeods upon te bond botwoen the aggregate andthe mora pase. For the sue coment cater, an ang aggegate Would require abigher water cement ratio than iegulr aggregate, which, inca, wll require abigher water-comeat ‘ato than rounded agzregat, Grading ofan agaregte: The pail ze distribution ofa agregas determined by seve aalysis js termed grading ofthe agregue. The pac saedsibton of asso aggregate shoud be such a she smaller particles il he vos berwecn lager parle, Forte concrete to be dese and workable when {cab the mina of cement, water and fine aggregate shuld sigh more an sient fil he vous in course aggregate; n ur, de comet pate sould te lightly more tan suena fill be vous i Ue fine aggregate. A badly graded aggregl requires higher wale-cemen oad hence resus Weaket once Toe recommended grading ini fc corse grep ae gven in Table 11 Table 1.1 Grading Lim for Coarse Aearegates Perceotge pang (6 mas for grade sgregne of comin ie | 2m ise Sm a 10 5 = = an set iso = = =a 3070 o5st00 io 100 mn = = e 100 a 193s as 2070 “oss os a °3 oi 10 tie 26 om = = Ss mu 18:383-1970 bas recommended cera limits within Which the grading must ie wo produce satiny sare We fin of ae ee roi, ih ace serie ex td peo ‘aggregate Th sands are generally dividd int diferent zoos scoring to the perentage sang te ese imron sieve 18383-1970 casi he snd into fut zoes 1, Il, and V so at ine age ena ‘essing 600 micron sieve in each zone does no overap. The grading links of four zones ate penta Tanks 1.2. From grading zones [to 1V, the fine aggregate becomes progress vey fic, and Ue Fao fc tron, agregat tobe ued in concrete shoul be progesivelyTosoce, ‘Tele 1.2 Grading Lint for Fle Agzreates TS see cngaion Pectge pang 67 maa) ‘Gaig “rang ‘Grd Ge Gone omit onety a 1 100 0 700 5 seit sito soto sive Be oe os ‘St00 io Bie i ae So 330 loo Soi "ie ise ase an mi00 So he Sao te 10 50 130m oto oie bie es ‘An aggregate considered to belong toa zoe in which its percenage passing the Qiu sew alls ‘nd is allowed to fll outside the ints ie for enber ees by not moe than atl of px cen Fax cosbed ‘sve sans, the permisibe limit on IS:150 micron sieve i increasd by 20 pox Cen However the Seer atfect the Sper cent allowance permiued above as applied to ober sew sizes In case fain aggregae, necessary adjustments may be made in grading bythe adion of sage size ‘egress without separating ln ine and cca constuents. The recmmended paling Unis se Seco Tabee 13. Table 1.3 Grading Lime for Alin Argrepaes TS eve dergsnion Prete (ym) pang fallin ggg ool az oom 10, a 2 on 95st00 00 nn 7s 95:00 27s tn Se S050 0” im ie ass {30m os i Uni weigh of eparegae: The specific gravity of panicles depenison te minefal content ad is wsely taken a 2,60. The bulk density, dha is, the uni mas er cubic mete, furteydcpsdsen te oltre gamed ‘ue degroe of compaction. As a rough suid, te bulk density of sand and gravel i oot 700 Lp Iiredcton Reinforced Concrete 123.Water Nae the mod inporant and least expensive ingen of concrete. A part of mining wate i ized ig (pe tydon of cept fom te biting marc in wich Ue et agatgace we eld as arc Tos Rardened. The remining wae serves asa vic been te fae and coat tine, ‘ies We conse workable, ‘he wae used for mixing and curing should be fe from ijriousamouus of deli materia Pouble waters pene considered satay foe mang connec 124 Admictures Admixurs othe mate oer than the asc ingreicas of concrete, ic. coment, water nl aggrgaie, ‘cde theconeree mix immediatly befor or during wing to ody coe men of snceli eee ‘ofthe concrete inthe fresh or hardened stat. The properties commonly modified are te re of hysiotce o sting lime, workability dispersion and air entrainment A degre Of conrl mw Be exercises to cae rope quantity ofthe adiinture, as an excess quantity may be detrimental othe propre of the ose 13 PROPERTIES OF FRESH CONCRETE, To comomically achieve the perfonnance requirements of hardened concrete, namely, arbi he fresh concrete shoul sant anumber of equremen rom the minangssape ll ictaren ‘laced in formwork and compacted. The roguiremens maybe summarind as olen |. The mix should be able wo produce a homogeneous fresh concrete from ingredients under the action of sing forces, This propery is termed mazabiliy. 3 Tne mis shuld be sable, in Uti should nt segregme doing transporution snd placing when itis SSL e forces dng tmdting operation of lied ate The dency to cea te minimized. 3, The mix shouldbe cobesive and sutTiceuy mobile to be placed inthe form around he reinforcement and cas tothe required spe, The property is termed flowabiiy of teh concrete 4. The mix should be amenable to proper and thorough compaction ito a dense and compact concrete wit minimum voids under the facilis of compaction avalabe at Be sie. This propery teamed ae actability of concrese, 5. should be able to produce a satisfactory surface finch witout honeycombing o blow holes Tis apaiiy is termed fatal. [These divene requirments of minabiliy, sability, ronporabliy, placeabilty,Nowabiiy, compact tility a0 finihabty of tes eonerete ae collcively refered to as workabilin. The wekabsy ot ah SMa stata composite property. 1s iat to define weislyal apt of he workaity se single _A number of diferent pli tests ar availabe for measuring the wevkability of fresh concrete, cach mewsuring a pticular aspect of The widely ase ets ar () Ue samp tet i) te comping ee ae iy tbe vee-be consistency testa (i) te flow test tof these four the sump et sprigs he most widely usc primarily because of he simply of {Besrpartu required nd the tes procedure. Ther isuorigidcorelaion betwen he workable oben SSmeasre by difercat est methods, Table L4 gives te range of eapecied values of wortablity ence g by diferen uss for comparable concrees, {6 Reinforced Concrete Design ‘Table 14 Suggested Ranges of Workabiliy of Conerte for ‘Some Common Placing Condtone Placing ondions Deseo woe Ya of wont “Concretng of shallow sxctons with Veryiow ‘20:10seconts, we te ine ioe es 025.040, compacting foe \Coneraig of tg reistored see Low 105 seconds, vo-bee tine Sone wih vibration = 020-085, comparing fice Conating flighty reinforced sec. Media, ‘52 seconds, wee ne ‘ions without viraon, of bevy o ‘igor scion wits vibration 085.092, compacting fcr + 25.15% sampfor20mraseareste Coneresng of beaiy infos Hien ‘Above 0.92, compacting fake ‘iow wast vibton a For smaller aggregates the valoes will be lower. ‘Toe workability of fresh concrete depends primarily 0 the characteristics of materials wed, mix ro- onions, and eaviroamental conditions, 14 PROPERTIES QF HARDENED CONCRETE The principal properties of concrete which are of practical importance ave those concerning its strength: stress-strain characierstes; shrinkage and creep deformation, response olenperalur vasitions perme ‘bility and durability. Of ese, Ue stength of concrete assumes a greater significance because the sength {srelated to the strctre of tt hardened cement past and gives an overall pctare of the quality of concrete. ‘Compressive Strength ‘The compressive strength of concrete i based on 150 mm cube compression specimens cued under standard {aboratory conditions and tested to crushing ata specified rate of loading at 78 days. The cbes are usually Jaded ata slow srain rate to reach maximum stss in 2-3 mia. The compressive suength depends oa Ue ‘ype of mix, the characterises of aggregates, quay of curing and age attesting. The compressive sagt ven by different specimens forthe same coneete mix are diferent. The cylinders and piss of aro of height or length to Intra dimension of may give asrength of about 75 to BS per cent ofthe eve sent ‘of nomial strength concrete. Based on its compressive srength the concrete i designated by diferent grades. The various pates of ‘concrete as stpulted in 15:456-1978 and 1S:1343-1980 are given in Table 1S. Inthe designation of the ‘eretemis, he leer M eters tobe ix and be number tobe elfed characteris SEN fy APES Imreducton'o Reinforced Concrete 7 “Table 1s Permatbe Stress ln Concrete Fernie res Nina Gadeot | Chusceite Compresion “Tensoa ‘veagebond severee_| Seng fs? Denon. | Dina on Painban io io 30 a 2 7 Mis iy 30 a 20 06 Mao » 39 So 2B oe Mos B a5 ry 3 09 M0 ie 109 ry 3 19 Mas 8 3 30 4% u mao oo Ho | ao a a Tow 17 The valoe of penne tear ves re gven a Tate ‘Noe 2: Bond str given in Table 5 abl be orese by 25 per cent for bars i ompesion. as Hotere ccs eet, yg fe 1120p aot conse Tee ee car opal pees Te Ce ee ei gh eck in design. 1S:456-1978 specifies a value of 0.7 fa, Nimum* for flexural sireagth for normal weight concrete, See ee eacnemecun named a Mi tare rae ert sn em cing msn ig pce aa ee eres ag a ay ae 2» qi ae is Dn Stress, MPa 0.002 0.003 0,004 Fg Streastrala carves fr coucete (nr species) Strain 8 Reinforca Concrete Design ‘eigh-totaterl dimension rato of 2, londd in wana compression na est conducted under ior re. ofsuan. Thecurvesarealmostinar pio about onal be compressive reg, The sina ie mann BuRis apronimatly 0002. The peak of uve for high sueaga concrete reively sharp bt fo low Seng concrete curve has a a op Ifa unform rn of sess is adopted, i will wt be posible 0 ola le detcending portion ofthe stress-strain curve beyond the maximum ses ‘The short-term static modulus of elasticity for Seuctralconcree defnng the slope ofthe tngca to he ses-train diagram may be ken a , = 5700 fg Nau? 144 Shrinkage ‘The contraction in volume of concrete as it hardens and dries i called shrinkage. The shrinkage strain are Independent ofthe siess conditions in the concrete. If restrained, shrinkage. strains can cause cracking of ‘coerete and will generally result in an increase of deccon of the stvetural member with tne. Tbe eal= ‘lations of deflection de to shrinkage are given in Chapter 12. Two types of shrinkage strains are recognized, ‘namely, he plac and the drying shrinkage. Plastic shrinkage occurs uring the first few bours afer placing the fresh concrete in forms. Exposed surfaces are more easily affected by expasure to dy air because of he large conict surface. Drying shrinkage takes place afir the concrete ha attained is full se ad a major ‘peda te hyration process in ube cement ge ie accomplished. ‘The shrinkage on frst drying is party irreversible and iscalled intial drying Shrinkage. the dry concrete ‘is resauraed with water an expansion (sometimes referred oa moisture movement) of about 60 per cea of ini drying shrinkage will occur ‘As a rule, concrete that exhibits high creep alto displays high shiakage. The magnitude of shrinkage Strain depends onthe type and fineness of cement, cement paste content, wale cone, ratio of fine to coarse ‘ageregate relative humidity, temperature and duration of exposure. 145 Creep of Concrete ‘The concrete under constant compressive stress shows an imunediate ssi followed bya further deformation hich progresses at a diminishing rate as ustrated in Fig. 1.2. The immediae sain is often refered was Deformation deformation Time ig. 12 Typleal curve for crnp in concrete subjected tow constant et Type cur eelpreaine tas ’ Inroducton Reinforced Concrie 9 the elastic strain and the subsequent ine-dependeat strain refered wo as creep strain, ot simply the creep. the load is removed, the part of te stain which is immediately recovered is called the elastic recovery aul the delayed recovery is termed the creep recovery. The elastic recovery isles than the inital elastic sean ‘because the elastic modulus increases with age; Ue ereep recovery ia smal portion ofthe total erecp sain, ‘The magnitude of creep strain depends upon concrete grade, enviroment and stress-ime histoy (ie. sires level, duration of lading adage at loading), Generally, the creep as Tile effect on the stcagh Of & Structure, butt wll cause redistibution of ses inreinforced concrewe members al the service loads and leads ‘oan increase inthe service lad deflections, ‘Toecreen sain canbe determined by deducting he elastic strain and shrinkage strain rom the total stra ‘According 10 1S:486-1976, inthe absence of experimental data the ulinate creep strain way be esimated ftom te following values of ereepcoeficint, which i defined asthe ratio of eeep stain Wo elas stain at the age of loading. eo oting eeepc 9 tae 2 a ae i 3 ig 1.46 Durability ‘Thedurablity of coneret refers toi ability to withstand the environmental conditions to which iisexposed, ‘without deteriorating over a period of years. The resistance to. weatherag, chemical attack, abrasion, (st and fie depends largely upon its quality and consdwent mateils. Susceptibility to corrosion ofthe sect 's governed by the cover provided and permeabliy of concrete. Tae cube crushing strength alone is no a rellable guide to quality ad durability of concrete. There i a need to emphasize durability in design and ‘construction of a concrete siictre in ems of the upper linit ofthe water cement ratio. «lower lait of be used ia mix esign is determined by the relation: f= fa + IS mfg + 65S here fa isthe characteris strength at 28 days, Sis standard deviation given in Table 1.7 and & is he . Matisteal coefficient. For the defintion of characicrisic strength given in 18:456-1978, &= 1.65, 12 Reinforced Concrete Design + Tile 17 Relationship between Concrete Mix Grade ad Soothes sori Tee Ee ss at 28 days, Ni deviation, | SS 3 # 3 8 : 8 : 8 g @ Gi) Selection offre water-cement ratio: Determine tbe water-coment ratio to obtain the target mean com- Dressive strength estimated instep (0. Compare this watercement ratio with maximum wate-cementfalio ‘Specified for durability in Table 1.6, and adopt the lower value resulting in concrete having ahigher strength tan required, i) Betinationef air content: Fo be maxis na sae of aggregate tobe wsedestmate the approximate amount of entrapped air from Table 1.3. Table 1.8 Approsimate Sand and Water Contents per Cuble Mere of Concrete ‘Sposa condiions | Nominal masimum | War conaal per | Propordon ofeand | Eaxappol a Wot ‘often Scape. | eben gt e-| Cai neeztely | se fesares Yemen 10 ome ° 30 Vorahliy “ B40CE 2 is 3 8 Wasccenentniiono3s| —° 10 200 m 30 Mortabiny= oso CF 2 ito 5 20 (@) Determination of water contens and fine1o-aal aggregate rato: Fo te desired workability, the quantity ‘of mixing water per unit volune of concrete and th rato of fine aggregate wo wal aggregate by absolute * Volume are estimated from Table 1.8 fr the folowing sundard reference conditions: (2) Crushed(angular) coarse aggregate, () Fine aggregate consisting of natural sand conforming to grating ove I of Table 12, ina saturied surface dy condition. (© Wotercement ratio of 0.60 and 0.35 for medium and high strength concrees, respectively (@ Wostabilty corresponding o compacting fair (CF) of 080. (©) Adjustment for any difference from reference values: Adjust the water coolcal, and sand percentage as Pet Table 19 for any diference in workability, water-cement rai, grading of fine agaregate and for rounded aggregate for Ue paticular ease from the referonce valtes. ‘Table 19 Adjustment of Values la Water Content and Sand Perceatage forthe ‘Conditions other than Reference Valet ‘Changes conditions plied for Table 1B ‘Adjriment apie ‘War coment er cen sands il see "eae For sand conforming w grade toner land V of ° TUS foraonet ‘Tabled, Ses 1978” 13 for nove ut 20 far goo BV Tncreate or decrease inthe alu of compacting fac] 3 perenat. ° tol ach 008 increase decreases wate semen ati ° 21 percent For rounded agregte = 15kpat = Tercent ¥ Imioduction wo Reinforced Concrete 13 Gp inf mrt mnie es pw eof en aetna, res fe a sieataet ts Cie rn an spt einen mine edt gen een a pe te Seite hearers tat an a cw san ay ctu aaa SminiSuted camera tee eae tne teas see eee ao es a em i etal as 1000 =1-[Eew] aw . whore W and C are masses of water and cement. respectively in kg, ad sar content in mandy, the density of cement iat per tite. For the top of ie aggrept to lta aggregate by absolute volume, the absolute volumes of coarse ‘ud fine aggregates per cubic mete of concrete Va and V,, respectively, may be calculated as flows VeepV, and Va =(~p)%, where ¥, ste absolute volume ofthe tal aggregate. ‘The mix proportions by volume ae: Water Cement Fine aggregate Coarse oparegate Wine) (€X1000 ¥9 (a: Vaca): Vu) ‘The mix proportions by mass canbe expressed as: Water Cement Fine aggregate Coarse aggregate Waive): Clk): Va XH 1000 te Vas 1% 1000 kg) Where Yq and ate the densities ofsaurted surface dy fine and couse aggregates, respectively, i kg per lire If no information is available y, = 2.6 kgie for uncrushed aggregate and 2.7 kg/itre for crushed aggregate may be assumed, ‘iid Trial mizes: The calculated mix proportions are checked by means of tial batches. The quantities of ‘materials foreach wal shall be suficient for atleast Uee 150 mim concrete cube specimens and concrete equied to carry out he workability test Normally, four trial mites ar required, The il mix n0.1 is used for workabiliy, The mix is carefully observed forte freedom from segregation, bleding and for finizhing properties Ifthe measured ‘workability difes from the sipulsted vale, the water content is adjusted accordingly and tix proportions fre recalculated while keeping the free watercement rato ale preselected valve which will comprise lal ‘mix no, 2. In addon, two more tri mixes nos. 3 and 4 shall be made with water conical sane af ial ti ‘0. 2nd varying the fee water-cement ratio by « 10 per centof the preselected vale, For ese two addional ial ix ns. 3 and 4 the mix propontions are 10 be recalculated for alered conditions ofthe free water-ealBBY oimy pee oma TTT (7 REINFORCEMENT ‘oneree i sirong in compression but weak in tension; thus see bars are used to reinforce the concrete to ‘ral tensile stresses resulting rom the applied loads, Additional reinforcernentis occasionally provided to 2ecessary for heavy lnds in order to reduce long-term deflection. The material tobe used for reinforcing Ue onerete must posses the following properties: () a high yield sirength (9), (i) ahigh modulus of casicity Ed itl) an ability t develop a good bond with concrete, (i) acoeffciat of expansion which i nearly equal ‘that for concrete, and () easy avalabilty. Stecl as reinforcement satistes all the above requirements, ‘Te reiting composite material is ideal cause asthe concrete ses, it contracts and thus grips the reinforcement. ‘This adesion maker concrete 1nd sce! to work together asa single material 7.1 Types of Reinforcement ‘ypes of steel used for reinforcement are: mild ste, medium tesile steel, hot rolled deformed bars, cold ited high yield siessdeforned (HYSD) bars and hard drawn eel wire fabri. In all cases the modulus {elasticity of tel shal be taken as 200 ENF ‘Steel reinforcing bars are generally round in crss-soction since they possess the maximum possible ‘rimetertohavesabererbond with concrete. To minimize the slippage ofbars relative tosurroundiag concrete, ‘as o« protrusions (projections) called ‘deformations are rolled onto the bar surface. "The lugs gencraly ave an average spacing no exceeding 0.7 § (nominal diameter of bar, anda height a leat O04 O08 ot (K.nomial bar diameter, alo they must present round at east 75 pe cen fhe nominal bar perimeter, Te ‘efonnations ae placed so tha the angle to the axis ofthe bar isnot les than 4°. Geacrally. tongitudinal ibs ‘alo present on the surface ofthe her. The nominal diameter ofa deformed bar is equivalent to that of 2 sain bar having the same mass per init length asthe deformed ber, The steel reinforcement is normally designated as grade Te250, Fe415 and Fe$00. The corresponding ‘eld strengths are 250, 415 and $00 N/mun’, respectively, and genealy have well defined yild pois, 700 igh yield strength steel oy bars 500 rang i inate stress Steel stress, MPa lastic stage 0 0.04 0.08 0.12 0.16 Steel strain Fig 14 Types stress-strain curve for tee relaforcement Intron to Reinforced Concrie 19 ‘and, finally ange in which the sires dops olf und fracture curs, {ihe ates at the yet point ore yield suength is avery imporant property of steel reinforcement. The ‘Bildstel, when used as reinforcement, ca yieldon overloading, resulting in anexcesivecrackinginconctee, thes giving sufficient warning before faire. Because of te eaty avallbiiy of bigh yield stl bas ae ss of mild steel is limited. However, du to its higher ductility i can be economically used fee soconiony ‘minforcement ad io blast-and earthquake resistant svete. {Tee mild tet bars canbe plain or deformed. The permissible stresses for deformed mild see! bars ae if same as fr plain mild sel bas excep for binds, wher te permisible bond res canbe Inctcaed by 40 per cea in ase of deformed bars High yield strength deformed (HYSD) bars: Toese bars are obtained by subjecting the mild eri wortng by tensioning and wasting, The process increases both he yc as well ase ulinewesength Stsoel These bars are extensively used in reinforced concrete works and have alment replaced mild seat [ere are some disadvantages associated wih the use of high eld atrength scl namely fxs a Gucy: [Be lower percentage of longitadna ste! in te beam reduces the capacly of ie section in shear widen ‘ighor sires level, the defection and cracking ofthe member ac lange, Table 14 Permisibe Steer Stee Relaforcemesit & of ie a i Te ie Ne Permisbe sveses, Ninn? Piso Pisinor | High yield] Wighyied sefomediid | | deforaea | ateteheet | steettia’y stelgrade bars] medium conte | Suc'Stgeais | SSS RSS) ‘a ease ee | Teen oc, @Uptosidtectadig 20m] 40 Hat toe guaran. | 230 2s Gore em 2. feed yee | Ba Ei i 00) Compression ia cotuma bars] 130 0 130 190 (ae oa (| Cipresion a barns tea orn tab when re] The calculated compressive ses inthe rrrouading corte muliptcd by ess fmexibe todalr rato or ee, whicves OSE Me “inne aot ro) Upwsndbcioding 20mm} 40 Hattie pcan. | 190 20 a if feed eidcres | 130 Bo Nove For Bigh ed sap cloned bso re FeS0, he pine wees GiSa watog MoT ‘coon sal e035, The permite tres or hes ual comprenioa mantecseen aaah eee ate 2 Foe nlded wire are confrmig to 1S-156-197, permis vale ia tension 230 Na” 5 Fortis poss, the ved ree of els for which ere um ceey asin anon aa ne a 4. Wate elem ge ite be pie enue le mda 90 pr coms ster of scl creased ar opcifed a Be, 9 Reinforced Concrete Design tag rai inns ch 7 el gn Tae sla nam tn ya tain ae SERGE GaGa se Ric meen ne tts u . steels lack a well-defined yield point: the yield strength is taken as the stress corresponding. SNE TGA Ane ee nota na ‘meat ty gels tee : Se net tpt vn opt wt rte angkncaee annie 8 FABRICATION AND PLACEMENT OF BARS he fabrication and fing of reinforcement bars are the other factors of majorimporance. The reinforcement ars sal be fee from lose scales, lose rust and coats of pains, ol, mud or other coating which may reduce }+— Upper column (End spanrirel beam Main bean Cross T Fomdation (footing) lg 1S Typleal slomonts of sructral tern Inuoducion to Reinforced Conerete 21 cr destroy the bond. Before fixing the bar in positon the for ar esl ale soa 1 facto ripping Sno ge ope sun. Ne Reinforcement shall be bent and fixed in accordince with procedure specified in 1S:2502-1953. The reinforcement shal be placed and maintained inthe postion showa ine strctrl drawings. The conere ‘over othe reinfrcement canbe obtained by using precast soppat locks, In de o maintain the ars position daring concreting, the bars are ed wih each ter using ether binding wire or ack weed, For sects ico pe Cm ible. Aer ing dee a off so hat dos touch ie formwork at is eas to using Of reinforcement, Ab 7108 kg of wire i requled for one tonne a'ec bas. > 1.9 CONCRETE STRUCTURAL SYSTEMS ‘structure should basicaly full four requirements, namely, structural, functional, aesthetic and economic, ‘icy it should serve its fonctional purpose while being safe, serviceable, active and economically cast ficient Alough most of the sictires are designed for a lifespan of 50 years, a propery proportioned ‘concrete souctare may havea longer use lie “Atnejority of the concrete tructres canbe considered toconsctf nme hasiesincural elements. broadly lasted tt beams, slabs, columns, wall and foundations, These elements shown in Fig. 15 are briefly ‘escrbed a follows 194 Slabs ‘Slabs ae te strcturl elements in flexure that ae subjected vo loads predominantly acting perpendicular to thei surfaces. The comunonly encountered flor slabs are horizna, 2-dimeasional elements that transmit ‘gravity live ad dead) loads to the vertical frames ofa structure. They can bea slab ot beams, a8 shown ia Fig. 16(a), a slab without beams (Ct slab or plates) resting directly on columns, as shown in Fig. 1.6). Depending upon the plan dimensions a sab may Uansfer Ue Toads in ove direction while essentially acting ‘ss one-way labor in (wo perpendicular directions with a two-way slab action, 292 Beams a “These are one-dimensional styctural elements in flexure that usually transmit the load from slabs to vertical supporting columns. They re nocally east monolthically wit lbs. They may form a T-beam. section for interior beams or an L-bearn atthe exterior of te building. The plan dimensions of asia panel determine ‘whether the lor sib behaves essenially asa one-way ora two-way sab 193 Coleras ‘These are one-dimensional srvctral cements subjected predominantly to compression na structural oor system the columns ave generally the vertical clement supporting beams: In moc of te practical cases ey’ ‘are subjected to both axil and bending loads, and ae of major importance in the safety considerations of ay Sire. fa stractural system consists of horizontal compression member, such amember would benonmally considered a heam- 26 Relaorced Concrete Design 23 DESIGN FOR FLEXURE, ‘Thischanes deals with te proportioning beams which ae primal subjected to exual action. The design ‘rinciples tat apply wo a simple bean can be extended for designing relatively cOmplicted cructaces ie continuous beams, slabs, staircases, footings, and retaining walls, Tae beam canbe of any shape: Homeves, ‘ectangular-shaped beams, being most commonly used, wil be studied in detail, The application ofthe working stress design procedure, explained in Sec. 1.10, leads to the following bade ‘assumptions required fr the derivation of working stress design expressions: (G) Plane sections before bending remain plane after bending, i, he strain is proporinal tothe distance fom the neutral axis . (@ Bows concrete and reinforcing stel obey Hooke's law, ie, stes-srain relations are straight ines under working loads, 2 3) The tensile suength of concrete negleced. (4) There isa perfect bond or adhesion between the concrete an reinforcing ssc! 0 that ther is no slippage between the two materials (2) ‘ne mova ion asthe value 28013 a) (rounded of tothe nearest integer valu in accordance \ with TS:2-1960), where 0, is die permissible srss in compression due to bending in Coneete te Nisa. ° (©) The steel areas assumed to be concentied atthe cenuoid ofthese, @ Te other basic assumptions conceming deformation and flexure of homogeneous members ae valid The extemal forces are assumed tobe resisted by internal compressive forces C developed ia concrete, ‘and tensile forces Tin see. The external moment ala section i resisted bythe intra couple formed dae touhe above two forces in concrete ating Uxough Ue centroid of wang distbytion of compressive css ro chon o resoant reste forces dEalcsion acting ae ensoid ots The deme Tee iow leer ant Sted by Hj, shows ng De iene svipt ine siesdstibton is ssc the neat axis passes ough Ue centoldot te ‘Hoa ers-scton forte condition C= 7 The wo tecuve hsed Beet Oe eos negli ats he fective aa of concrete he area of concrete are Oe metal ale the masa sess oth the cone andthe enfin el seach the pers alos ate sue in, hen the section i tmel bance econ Te eee fi section sealed te balanced er erveal nara ass eis dep blow be ep Compress deed ty ‘The tlanced design ans hat here is exactly enough renfrcement {0 develop th yemisie cormpesive sues i nce, I Wee leser amount of sel hen concrete compeaine aerate an be develope, corti suessed bl the poise ni andthe scton hae setgoread Ue such acuaon ite flue ofthe bam wl be ned doe versie tte The sant oe a vee sulicient warning Netore he al elapse of este Erte ene apa assed ee above the critics neu ae te selmi Jr wes ore el he tht quid fr te balanced eto, We seo is overejrce Le, the ennisible ve wile the maximum compose sce a ever ees iepenmisbe init The falar wil be intated dco overseas inconrete Asc eer loeb, ‘be suture fs suddenly witht any warning. The acta ear arses slow i cd maa ae Ins talanceddsia tough bo maura sree otc permis values tisnonacecety tx mst economical tion fo a design inthe penest einen i moa of tnderenfrced scons. The ref under reitoreed mere a reduce. indersuetsd mener arc deepr, tle and face lesser dleton publons th the sallover member Working Sess Desi: Flere 27 2.4 RECTANGULAR BEAMS REINFORCED IN TENSION ere sae tent etc nem emit sac a ine acetic Suneeae lime pana ic gueeiennacehe cotker hand, desigi’ of a beam for Mlexure consists in proportioning the dimensions of cross-secd qeatenciorata naman iene 24.1. Analysis of Beam Reinforced in Tension Only ‘This type of bean is commonly known as singly reinforeed beam, Egure 22 shows the assumed disttution of strain and sess across the rectangutar beam secon withthe following symbols: S'ae —* Actutl compressive stress in concrete under bending y= ctu ses in stein tension & = sninincocrete = 04/6, = salnia deel 4 EqE, = moduli ofeasiciy for coneree and ste, respectively mm” = modilarraion B/E, - Be with ot bean 4 = effective depth rom exseme compression ito te cent of tensile enforcement X= dep of neue axis from extreme compres fib £ "= leveranm, te dstince between centroid of mpressio fore tothe eeatridof tensile force f (a) Bean (b) Crwss- * (@) Stress section distribution Fg-22 Assumed tran and ste debate na bam ejected to ending moment {Fram ue stain dition curve: ' Sf SelB) x eae Jae +8 28 Reinforced Concrete Design siving, adil +0 fn g) 02) ‘and bence, f= (+o Jmo where kis the neutral axis coefficient ‘Since the centroid of compressive fore sata distance of (13) from te top edge ofthe section the lever ann is given by oe ‘gad (3) = (d~ bd) = dO. ~ WB) jd 23 were (=A), andi termed the lever arm constant. en “Total compressive force on the secon, C= 05 og br es eo ‘Total tensile force, 7 yA For the equilibrium ofthe section C= 7. Moment of resistance: The moment of resistance of a renferced concrete section, M, i equal to the moment Dfteccupleformedby the otal compressive fore (C) acting ae centroid ot tae compressive seas aya, fd tu leaile force (7) in stel acting a the censoid of reinforcement. Thus he moment of resistance of ‘ection with respect vo compressive force: M,= C205 4b (d= 213): en “Moment of resistance ofthe section wi espect othe tensile force: 2s) My=Te=(@,A)(d=23) “The allowable moment of resistance ofa concrete section may be computed from Eqs. (2.7) oF (28). tis governed by te penmisible stresses in both concrete and ste, permissible ses in stcel or permissible sess concrete depending on whether Ux section is balanced, under-reinfoced or over-rinforced, respectively [A section is tenned balanced wen maximum steass in concrete and steel each hes permissible vals a De same time, Le. dq, = Ogg and = 0. The neu axis fr his case is called erica nexiral ais ad is denoted by x as lasted in Fig. 23 (a. Ths from Bq, (2.2): : Kel +ojmon)= hd oa “The comespondiig value of the moment of resistance with respect vo compression s obtained from Eg, 2.7) M,=Ce=OS 0h) (dh) = DSagbuA(t~/3)=(OS0 ghd = Ob? 219 ‘iscated be momento esistanee ico fr atlancd rectangular setion, The ment of resistance with {expat tes force fen at : MaTra(eA)d-Ka’)= (aid anf ‘Area of reinforcement can be obained from the condition, T= C. ! “Thus, fora balanced section 1 1 180} allt)= Ob kd ean A= SaK bli) Working Stress Design: Flesure 29 TI i 7 4 Pcriticat nesteat =f -— aD ‘exis ~ ~~ ~ a fe / “A LS / / . a) ae Cae Rone one ofa ope/a (a) Balanced section (b) eee &) eee ig-23 Typer of singly reafored sections (orm tren dributon In a relaforced concrete setin) Lethe percentage of renforcing sel be expressed as Di Ay 106d, then fore balanced section fom Eg, (2:12), Pow = WOE) = (50K 9,.90, aus Design Constants for a Balanced Section For MIS grade concrete and Fe250 grade mild see: 0, $ Nina’ y= 140 Ninn nd m= 2803 fac = 5 Ninn c= 140 Nini, and m = 2806 0, 18667 (ay 19) (ode of the nearest inege aac) Newt i 2 ‘and thas J =1-GA)=1- (04080) = 0265 Q 05 0qck,j #055 x0A04 <0865 = 0874 Ninn? Pons = 50, Oty 50% 0404 x 51140 = 0.721 ‘The design constants for some ofthe material ace given in Table 2.1 ___ ie amount osx provides than ti requ fr he balanced ection ism ts penile ve wee cocaine. ie cau ove ‘upward (x <4) and soch a section is called under telnforced Foran sry ale underelnreed sete, a8 shown in Fg. 23 (0). Foran Reps 320 Reinored Concrete Devin contents for x Balanced Seton atten Fornea 9, = 20 Nin Gaet | Perinie | Male Gente | rain tee |r ‘canna _—. ran ee ee ee a aan poms | om | 9689 | 0 Mio Se [| Sem | Gas | Use| es | 0a | oss | hte | oe Yas 475 } 1. a vs | a8 | | Sas | Sh [ ta | 3s | oa Looe De Noo i od : Tz = Agoda ~s13) and on) io, 05be) ett one ton et set rotcicmretn ae ae eshe Catena Pease He i A smile c istered oe reson ie eae bs eBnlevale. F veces wt Tac eettuSie id Pe piowatte M2 C5= 05a 29) 8 ain = O50gdbr Ae ee sect ca sition of the neutral axis ( «ie, h dan Aging mown, poston oe nena Ferg eit wate os om ees Erbe Dao bite had a= Aum Gall DE “ bea 2mAgr-2mAgd=0 Re oe elo mash % yo ye Sle) i a : Subsuuting AJM =P. weRE Ae ( neomteal ( 7 amp +(nip?+ 2m) ; sive sig only taken since Reanat be nea as vente cones nly ence inte mo SNA senion on. fa Beam Section —pateminaton of Moment of Resize an ood-Cart COPS Of aia pe rt er duane A) mam SS eae) meen “The steps involved Fe of 1b, d and A, trom Eq. 2.16). () Find he position of atu 1 evil ants from we known values of Working Sees Design: Flesure 31 (Gi) Find the position of erica neural axis for de oa ral axis, forthe known pennssible stresses in concrete and steel (2 Cony is dain e ye fa toutes. te censor ONC beset wrod (@) Calculate the moment of resistance fo the ap cer tr ponine poet M,= 05 Oa, bx (d~ x13) forte overenfrced secon ‘ame M,=04Aq(d~ x2) for tbe undr-reinforced section (©) I tbe effective span and tions Iie elective spn and sro ctdions of te team ae now, comput the loaning Example 2.1: A reinforced concrete beam reinforced with 3 bars of 0mm ¢, a shown a i stresses induced inthe materials Is HYSD stel of grate Fost nay sean ih ss itn tae om oe ee soe lass Nacsa ogee F | whit length i= 5a Fi 24 Design ofa ingly reinforced concrete section ibjected toa bending moment le effeciv span be above beams Sun determin esi lade cry fy Te wt weg ofc ny sabes noi Penmaes M20 grade cone FSIS peel a Oe 7 Nando, Modatar rio, m= 280/304.) = 28043 x 3.33 (say 13). Nex, ‘Aq= 3x7 (20)'= 942 mm? (ateruatvely from Table CL, Append C) 4, Thus, py 9a. 0.004486, and mp = 0.05832, 700 ‘Te cece of pho atl mest acs = + 2) — mp = 0288 sinh Smet eho te cal neal axisish= Mt + oo) #2300137 : vereinforced andthe concrete will each is maximum pennisible value fst Nea ge = Ga 2 Nt The dep of the actual neutral axis, = kd = 0.288 de £5 k= 0.288 700 = 201.60 an, Thus the lever arm, = Gd ~ 4/3) = 700 (201.68) = 632.80 men. 132 Reinforced Concrete Design ‘oe momeat of sistance M, = C= OS 6qb0E 2087x300 «201668280 2133951104 Niam Sireses io mater Ga.= Naw n= Chg (O5 cub, To5i7 x 300%201.9962= 2471 Nom? Ta30Nina? (+00 _ pearying capac: Let w AN be iit od on toned n= BP Bane MoM, Tht 25 w= 133931, giving w= 42861 etn etre ever to rfrcoment be SO: then he toa dp of ean aide 30-10 50 7500. 25 = 5.6254Nin Seeweght oben, y= (0300 0750x1025» 5.625 a nn 4286 8625 371285 WN 37 KN Superimposed lade Dem an sng we the bean per mete run. The maximum bending : meneame ge 2 a ms se ge a om cnananmanacra atest Coapression side (@) cmse-eection (9) Eiteerive (ey Tematorned section (8) Stress antares cfd cones econ wider ae ‘Berure after cracking rateratively by sing Eq. (2.16). (i) Determine the lever am, z= d~ (3) Me Gi) Find We tess induced inthe materials due to applied moment M: ares in steel, = MICA) Hoes in concrete, ge = MICOS B22) ‘or altermatively from the sess iagran ge" (lm) xKd~ 2). Ot ——vOvOoe eee ne Working Stress Design: leave 33 Example 22: A reinforced concrete team o rectangular tecton having a wd of 300 mm ae overall epthof 750mm sreinforced with 4barsoF25 tam gon asion side tan efccuvecover of 30 mum, Determine ‘sZamunom sree ce incense ton ‘Sabjeced oa amen of e0 EN, “The concrete used is of grade M1S. sean tension sl when edo eee Solan Pe gre conte: @u.= Nin end n= 2810-04) Cuesta popes b= 300mm d= 750 50700 mm ‘Area of tension steel, Ay = 4 x (wd) x (25)? = 1963.5 ma*, odcermne be depo Be cura anit ke mone tere some wes shut BeUA 8.667 (ay 19), bxen=maid-0/ 30 x2 = 193 1963 5 x(700-) P+ 24871-17407 =0, giving x= 31103. stomatvey = = hd wbeieh = (n'y? +2 mp" pane Imp = Aalids 19% 1963 S100 % 700) = O176S Tnetefore, f= 0.4443, an ‘= kd= 04443 700 311.01 mm, Leveram,2= x3 = 700 G11.093) 25963 mm, Sraser ‘Suess in te, = Mid = (15010919635 5963)» 128.11 Nu Suess in emer = MI0.56 2) (150 109K0.5x300131.03 x 5963) «5.39 Nous? Averett te res disthaton agra: af 0-Jm)=2Kd~2) aa. 31103 m a=z)" 19 Go0~311.05) 539i of 242 Design of Rectangulae Beare ‘The following types of problems are generally encountered {Type I: Determination of Tensile Steel ‘The cross-sectional dimensions ar fixed from architectural or other considerations. The materials tobe used ‘and the moment tobe carried are known, The seps involved ae: © Computing the design constants m, kj and Q forthe given materials. (© Decaeining te allowable moment of resistance or moment caying capac Of he set, M = Gi) Comparing the momento be resisted with M, obtained ia step i). The following cases may arise Case I: MM < M, te section is wo be designed as an under-eenforced section. Take moment about the neutral ais, Then, we have © bx(alt)=mAfd=3) oF Ay=bx1amd 33} ain Since he section i under rinforced, ° e 3. Relyjorced Concrete Design MEM =O Add) sren(e-3) eae Ps solve fr and hen for Ay (Case 1:1 M> Me ects ver eit then Mati =6Scbs\d~ G2) Fist detemin xand hen A fom the equa ote by equating th momento stoned arcas sont tne mAy (d~ 2) = bx (W!2) oF Ay=bx'72m (d— x) Howeve, 164561978 doesnot pert ue of oerenfred section, In such css iti preter to desgnitasa doubly nforcedbean where rence ise roviedincomprsciontag eae Sgt te coor Example 2 A recngulr bam of sie 250m 1m depth ovrl iso cary a. inant of S0 aN rn ain a a at un, Te macro be tsd ae MIS grade concen igh laze formed scl prac Fols {We design moments 40 Nm ised, decmine the amount of steel requ. Soluion. FocM5 ge conte and FoAS grade stk San SNin oy 220s 19 ot 0599 (Goan Tab 2) Bor0mm, desu sveesomm (ical moment of ssance: ° Mym Ob o.689 «250450 = 33.36% 10° Nam = 33.364Nm. ' om =s08am Since M,> Mb ecto isto be designed as he und fred section Mya gha(d—223)= BOA, (d~ 22) = 30x 10¢ Aud ~ 2) = 391,304.35 oF Ay=391,304.35(34— 2) a ‘Take moment of ansformed areas about the meatal axis: x)= mA(d-%) 250272 =19 Ay (dx) Aun6579.2d~ 2) ® quate (1)and@),* 6579. 2Md~ 4) = 391,3043534-2) 6579.2 Gd x)= 391,30435 (dx) whered = 450 mm YB 13502 $9,478. + 2,676,5003 = 0, Solution gives, x © 125.54 mm, Substituing this valve of xin (2) ‘Ay = 6579 (125 5471450 ~ 125.54) = 319.57 mv Provide thre bars of 12 min 9 (Ag 339 mm?) Gi) M = 40 kN, Inthis case since, m support uniformly distributed 1nd of25 kN inclasive fits slf-weight, The width ofthe beam is restricted to 250 mam, The concrete wo be used is of grade MIS. Determine the effective depth and amount of mild steel tensile reinforcement required. ‘Solution. Por M15 conérete mix and mild see eiforcement: Cae 5 Nan’, 6 140 Nfzun and m= 19. ‘The section will be designed as a balanced section, The design constants ae: k= ML + (6q/mou)] = 1 + (14019%5)]=04 Jal ~ (W3)=087 O05 Gq kj= 0.5 x5x04x087 =087 Nina Thus, M,= Obd = 087 bd. Maximum design moment: Ms wR =25x0.5)= 78.125 Nan. Equa M, to M, 087 x250x df = 78.125 x 10, giving d= $99.33 mm. ‘Adopt 22 mum ¢ bars. Overall depth: D d+ (half diameter of bar) + clear cover = 599.33 + 11 +25 = 635.33 mm (cay 640mm), Ertective dep, d = 640 — 11-25 = 604 wm. Amount of reinforcement: ‘Aa = Mi ff) = (78.125 x WPYC140 x 087 x604) 2106195 mm? Provide 3 bars of 22 mm 4 (Ay = 1140.30 ma’). 2.5 RECTANGULAR BEAMS REINFORCED IN TENSION AND COMRESSION In practice, very frequently itis desirable o even mandatory to have a section of restricted depth in ordet to ‘oimply wih some architectural or structural requiremeats wherein the section has to carry 2 moment ‘than it can resis as balanced section, Ifa section is aritrarily made shallower Usa the baled desi the resuling over-reinfrced section the concrete is oversiressed, while ste! is suessod to is permissible value, When this condition exists, the allowable compressive force can be increased by providing reinforcing Stee inthe compression zone, as shown inFig.2.6, Such section reinforced bot in tension and compression Working Siress Design: Flere 37 ‘is knows as a doubly reinforced section, Doubly reinforced sections are also provided inthe cases of load ‘where reversal of stresses may take place, eg, wind and eartguake loads, The doubly reinforced sections are recommended only when they ae absoluely necessary. In reinforced conerete structures the creep deformation of concrete produces an. adiiona st nd pay ines te Sr veh cine ssc To yaaa Te Fala. 15-456-1908 si te esos reinforcing bars ina beam or ‘When the compressive resistance of tie concrete Is taken inlo sccoun, shal be taken as (LS times the Oe aw 138. Relnforced Concrete Design ‘Equivalent concrete area resisting compression is eh p Agm(Ox-AJ+ 1S mAgebr+(Sm- DA, 17 220) Moment of resistance: C, and C, ate the total compressive forces in concrete and sie, respectively, Take ‘moment about tension stl: (ome M=Cld-n1)4Cyd-2’) = bx(0.50,) 4-113) (15m DA laa dd) =, 98, ea ‘Thus, in the design of doubly renférced beats, the moment of resistance is assumed to consist of two ‘ars. Fists the resisting moment M, ofthe beam section with only tension reinforcement Ay forthe balanced ovcition, as itusirated in Fig. 2.7 The second is ube moment of resistance Mf an auxiliary section due to the couple formed by the additional compressive fore in compression stel A, and Ue tensile force ina lke ‘mount of additonal tension steel An qa a app r hE a ([% Si % " Age Sst, a stg — adese a ost Astz Hh = HhanQbat Mo MMe bos (2) Section (b) Singly reinforced — (c)A, singly | seg 0 balance compression steel A, Fig. 27 Analy of doubly relafored course cation ‘The corespoading values of tension sce are: ag = MyM). ond Ae= Myod =4°9 am oval tension sted, A= Aus + Aus ‘The wea of compression steel can be obained from Eq. (2.21) or akematively by taking moment of ‘wansformed areas of compression sect A, and additonal tense see Ay about ie neutral axis, (Sm - DAG -d")=mAgld 2) A= miAg(d ~2¥{(.Sm ~ Nox 49) ez Tue following (pes of problems are usally encountered in practice in the design of doubly reinforced concrete sections Type I: Determination of Allowable Moment of Resistance or Load-Carrying Capacity 1n this case, eros-sectiona! dunensions, area of reinforcements in tension and compression, and grades of| materials used are known, The steps involved are: Working Stress Devin: Flexure 39 (9 Determining the positon of the neutral axis by taking moment of transformed areas shown i Fig. 28 about the neat ax BEM) + (1S m~ 1A, 2) = mAgid-2) be fe SmI) ge xt pte Ase (a) Stress distribution (b) Cross-section. () Transformed section Fig.28 Transformed area of «doubly reforeed concrete ‘sein after cracking (Gi) Deternining he depth ofthe critica neural axis x (Gi) Comparing x wit x, to determine te sizes level inthe two materials. The following wo cases arise: ase (a) IE.x> x, the beam is over-reinforced and concrete reaches its permissible sre, ge The Mis tiven by, M,=050qhr(d ~119)4(1.5m ~ Io ~€’V8h dd) Case (b), 14 <1 see reaches its permissible stress og cartier, and M = (bx12\(ofm) [eK 1} (4-113) +(1.5m = DA, fom) shld alsa Ved). (iv) I the eftecsive span andthe support conditions ofthe beam ae known, compute the load-carrying capaciy. ‘Example 24: A reinforoed concrete simply supported beam of rectangular section of size 300 500 man fective i reinforced with 4x 20 mm 4 bars a tension fae, and 4x 14 mim bars at compression face a an, fictive cover of $0 mm, Detesmine:() the moment of resistance ofthe section, (i) the safe waiformly istributed load the beam can suppor in adtion to its sef-veight over an effective span OF 8m, ad (i) (Me ‘maximum suresses developed in concrete, tension and compression sens. ‘What will be the effect of motiying tension steel to 416 mun bars? The materials used are M20 grode ‘concrete and HYSD reinforcement of grade Feds, Solution. For M20 grade concrete and Fe41 grade see: ae = 7Nin", = 230 Ninn mm =280G.0q.)= 1333 (say 13. a5 per 1S code), Case Ay 4x 20mm §= 1256 mm? Ae = 4x 14mm 9 =615 mm, ‘Teansformed area of compression steel = (1.5 m = 1) Ay = (15X13 — 1) x615 © 11,3775 min ‘Transformed area of esonstel = m Ag 13 % 1256 = 16.328 min. Letxbe the dep of the neutral ais. Take moment of ransformed aeas our the neural axis 300 402) + 113TTS (= $0)» 16,328 (500 2) 4 184702-58219.17 20 siving x= 16501 mm, and B= xid= 0332, 40 Reinforced Concrete Design ‘The coefficient of eiical dep ofthe natal axis: A= {1+ ogim ig) = Ul + 230403 x7)1= 0.283. Since k> ky the section is over reinforced and concrete wil reach its permissible value. Thus Ogee 7 ‘Nias and stress in compressive tee! = 1.SmxIGqf 4981 = 15x13 x7 (16601 ~ 5016601 ‘ures in tesion steel C= mdqld—0E = 13x 7 ($00 ~ 166 01916601 = 183.08 Nim. “The allowable moment of resistance of section is 1M, = 05 0qDE (4 ~ 223) + (15 m= 1) Ay Oa (= 8] ~ 4) = 05575300 x 166.01 x {500 - (165.013) (1S «13 ~ 1) x 615 «7 x (166,01 ~ 503/166.01] (500 - $0) ‘= 77,509,487.96 + 25,044,858.09 = 102554 10" Nunn = 102556kNe, 1 the fond om the bean is w kN, then the maxisnum moment M = w x88 = 102.554 giving w= 12819 Nin. Self-weight of beam, w= 0,300 x 0.550 x25 = 4.125 kNim. ‘Superimposed uniformly distributed load (w,) the beam can suppor safely is Woe w= Wy 12819 ~ 4.125 = 8.654 KNin, Case is Ay= 4% 16 mn 4 = 804 mn? Acad 14mm ¢= 615 min’. For he positon ofthe neutral axis, ake moment of wansformed areas about the neutral axis: bxGM)+(1Sm—-DAy (ed) = mA, (d= 15022 + (1.5% 13 ~ 1) x615 x (&~ 50) = 13 «804 (500 ~ 2) 9539 Nin Bo 145532- 38602520 ving x= 13682 mm and k= 2d» 0274 “he vale of kas detenined in Case Tabowe is = 0283 ‘Since ky the section i wer-enfored and ess in tension tel wil each its pease vale. ‘Thos of 0, 290 Naw and compressive srs in exteme bre of concrete trom the sires gram: Gy x 230, _ 136.82 2 vw nda” 6 ONO Swescin compression see, 6, 1S me(@dm)(x— Md ~ 3) , 15x13 (230113) x (1.368 ~ SOMO - 13682)} = £2.47 Ni Tre allowable moment of resistance ofthe section is M, = 0SGq:bxd~203)+(JLSm) (1Sm~ Ig 4-4) Cue = 05 «6,67 300 x 136.82 (500 ~ 136.8229) « (62.4715 x 139) X(U.$ x13 ~ 1) 615 x (500 ~ 50) Working Sess Design: esure Type is Checking of Adequacy of Section ‘Thecrosssectional dimension aw afeas of tension and compression secs are given Its required to check ibe aesses developed inthe materials under the action of applied moment. The steps involved ar: (Find the postion of the actual neutral axis. i) Equate moment of resistance of the section interns of compressive stress inthe extreme fibre of concrete 6/0 the applied moment M and determine a. iy Compare sesse in tension and compression stels, ‘Exampled.7: A reinforced coacretesinply supported beam of rectangular section of size 300 x 500 mm fective is reinforced with four 25 mm @ mild stel bars on tension sie and four 20 min mid stel ars on {be compression side, as shown in Fig. 29 (a. The effective covert compression sel is 40 mun from the {op edge The concree used is of grade M20. Detemine the stresses i tee and concrete, ifthe beam ears ‘ranifonnly distributed loxd of 10 EN/m over an effective span of 10m. 00am, re | - rr ee] (A.Smt) Age a |awane| | Yaar |i . A | 425 ame iS ——+ See a dex san Cs { : Fe. algae dnt aired cme on Solution, For the beam section: b= 300 mm, d= 500 mm, = 40:mm and m= 13: twansformed area = m A, = 13 x 1963 = 25,519 mm? ‘Age= (420 mm g) = 1256 max “Transformed area = (1.5 mt ~ 1) Ay (15 x 13 = 1) x 1286 «23.236 mun Maximum moment, M = w P78 = 10 x 10" = 125 kNan. “To locate the neural axis take moment of tansforned areas about the neutral axis 300.72 + 23,236 Ur ~ 40) = 25,519 (500 — 1) 4325.03 4 912996=0 ving x= {80:52 mm. Leta” fibe the compressive tres in the extreme ibe of concrete in compression: then stress in concrete tthe Jevel of compression ste! O'ae= Ou X (180.52 40)/180.52 = 0.7784 og, Niu”. “4 Take moment of compressive fores bout the tesile steel: Mi, = OSc'q:bx(d~ x1) +5 ~ DAgd'a (dA) = 105x300 « 180.52 x (500 ~ 180.52) (1.5 x 13 ~ 1) 1256 0.7784 (500 - 400} oa, = (11,909,626.48 + 8,319,975.10) og = 20.229,601 58 Fac (4x25 man 4) = 1963 mm’, and 42 Reinforced Concrete Design tom 20229,601.58 6/4. = 125 10" aiving o4.= 6.179 Non? Suess in compressive ste, = 1.5 m O"ae = 15 x13 x07784%6,179 = 93.79 Nin, ‘Suess in tension steel 0G gd ~ x= 13 6.179 (500 ~ 18052918052 ‘ = 142.16 Nim. quating, Type it: Design ofa Section Design ofa section fora given moment and cross-sectional dimension invelves the determination of areas ‘of tension and compression reinforcements. The steps involved ae: (Computing design constants forthe given material, Assuming concrete cover to compression see. (Gi) Detemining allowable moment of resistance of the section shown in Fig. 2.7 (6) aS Muy (= M,) = 0. and comparing it with the design moment M, The following thee cases may arse: ()M= May (OM < Muy and (C)M> Ma, Forte frst 90 cass, the section is designed as singly reinforoed beam as discussed eater. ln the third ‘ase the setion is designed ak doubly reinforced beam, (2) Compatin the area of tension reinforcement Ay; for esisting May, Ags = Mu 2). (©) Computing the additional moment tobe resisted by intemal couple system formed by additions tensile steel A, and cemmpression stl A: My=M= Mg (x) Calculating additonal tension stock A= OM ~ Mylo (da) nd Ay= Aut Asa (i) Computing area of compression stet from the equation (~My) = US m~ 1) A Gan le = EVE d= A). [Bxammple 2 A reinforced concrete rectangular beam of size 230 x 450 mm overall is to support a uniformly disuibuicd load of 21 kN over an effective span of 4 m. The effective covers to tension and compression ses may be astute as SO mm and 40 an opel. Design ne beat ar ad ‘re MIS grade concrete and tild steel reinforcement. meen Solution. For MIS concrete and mild see enforcement: a= SNinm?, og = 140 Nini? m= 19, k= 04, CCros-tectional prope b= 230mm,d=450— $0 = 400 mm and = 40mm, Maximum moment, M = w £7 = 21 x48 = 42 kN. For balanced section, Muy = ( b = 087 230 x 400? = 3202 x 10° Nanm = 32.02 kN, ‘M > Mu the section ist be doubly reinforced: fat = Madu jd = 32.02 x 10'K140 0.87 400) = 657-22 ma? Depth of tie ential neutral axis: hd =04 x 400= 160 mn, ‘Aadivonal moment, My = M = yy = 42 ~ 32.02 = 998 KN, Comesponding Aqs = 998 x 107(140 x (400 ~ 40)]= 198.02 mm? 87 and Q = 087 Nin Si f Working iets Desig: Fleare 43 Au = Melo g(d- 2 where Fy = 1S mE) 655% et = (1.5 x19 ~ 195 x (160~ 40160 = 103.125 Nina’ ‘Therefore, Ay, = 9.98 x 1O7103.125 x 400 - 40) = 268.82 mun Hence, Ay = Aa + Ayy = 657.2 + 198.02 = 855.24 min, ‘Ag = 268.82 mun, Provide 3 bars 20 mum 9 (Ay= 942 mun?) as tension reinforcement and 2 bars 14 mm § (Ay = 308 mun*) ‘compression reinforcement. Example 29: A reinforced concrete beam of rectangular section hast carry a uniformly distribute load ‘of 8 kN/mn over an effective span of 10 m. Design ue section using M20 grade concrete and FosiS grade [HYSD steel, when: (9 there ino restriction onthe sizeof the beam section and (i) the sizeof the section 8 restricted t0'250 x 600 man elective. Solution For M20 grade concrete and Fe415 grade seek ae = 7 Nhtun®, Og 230 Niwa? and m= 13 2835, )= 09085, x,= k, d= 170.1 wun, and Q= 0.8985 Nia, Design moment, M =w P78 =8 x 1078 = 100 kN. (Design the beam suc that a singly reinforced balanced section i obtained, i.e, Mu=O.bd=M D585 bt 100.0. Assuming d= 25, d= 204725278, Therefore d= 652.84 mm; 6 = 652.8425 =261.14 mm ‘An overall section of ize 260x700 mn with efectve depihd = 655 mm willbe adequate, Areaftesion * renfortent for hissing enforced beam is given by: ‘a= Mig. jd= (100% 1091230 x 09055 x655) = 733.6 ma? Provide 3 tars of 18 man (Ay = 763 mun? fr easion eafrcement (Gi) The section s resid to 250 x00 mm eecive. The allowable moma of resistance of the singly reiaforeedtlaned secon i Maa= Ob d= 08985 x 250 x 600" 10*= 80.865 Nan. nce M > Myth beam sion is tobe doubly reinforced The area of sel Ay resisting the moment Mis Obained a= ‘a ™ Muu J) = BOBES x 10°/230 0.9085 x) = 647.13 a ‘Nteratively, Aycan be computed fom the condition C= Ti. ‘hay = 05 7 280% (02835 600V230 = 647.12 ‘Additonal moment M, (= M~ My) sessed by aiiona tension tel Ay and compression tcl Ae ete elective cover othe compression sel be 40 mun. The, Aa M~ Malloy @- 2) (100 ~ 80.865)» 10171230 > (600 - 40)} = 148.56 mu ‘Tota area of tension stk fet At A= 19568 ma ‘Arex ofcompeesion see: Aen = Mad eld =A) where Og (15 m2) Oa Oe — Age (1.5% 13 ~ 17 «(47001 = 4091702 = 99.05 Noun 44. Reloforced Concrete Devin ‘Thus, Aq (100 ~ 80:865) x 10°299.0 x (600 ~ 40)] = 34498 on Alteratively, compression stcl can be computed from the equation obtained by equating moments of ‘vansformed areas of compression and tension sce about the neutral axis: (Sm— DAG - Da mAgd-x) Aus 13 x 148.56 x (600 ~ 190.1M1(.5 x13 ~ 1) (170.1 ~ 40] = 344.96 mun Provide four 16 mim © (A, = 804 mm’) bar as tension sel and rwo 16 mun ® (A, = 402 mm?) bars as, ‘compression steel 26 FLANGED BEAMS ‘nmany reinforced concrete structures an, particularly, in floor systems, a concrete slabs cast monolithically ‘with and, connected to, rectangular beams. In such construction a portion ofthe slab above te beam behaves structurally spar of th beam in compression, The sab portion i called Ue ange aed Deu the we. Such ‘abeam with lange and web is termed flanged Beam. Panged beams may be Tee beams or Fl/heams, x0 called because of tir resemblance with the leters, and inverted L, respectively. The flanged beams are shown, in Fig. 2.10. The terms 6, by, D, and d represea the with of lange, breadth ofthe web, thickness ofthe ‘ange (ie. of the slab), and effective depth of the beam, respecively. In the absence of more accurate ‘determination the effective wid of the ange, b, maybe taken as sxpulted by 1456-1978, The values are: For T-beam by= (W/6) + by + 6D; or L-beatn: b= (12) + by + 3.95 em ‘where ly= distance between points of zafomonieasin the beams. This is equal othe effective span fr simply ‘uuppoted beams and is 0.7 tes he effectve span ofthe beam for continaous beams. However, the effective ‘Mange width in wo case should exceed te bread of web plus blf te sum ofthe clear distances to ube adjacent ‘boas on ether side, Incase of isolated beams, the effective Nange widh obtained below shall, in no case, be greater than the sctual width ofthe flange For T-beam : By Dy + ib) +4) FocL-beamm :by= b, + (05 LM{Q/0) + 4) 225 ‘Asin rectangular beams, the langed sections may be either singly or doubly reinforced. 246.1 Analysis of Flanged Sections with Tension Relaforcement Neutral axis: As in the case of a rectangolar beam the frst sep in the analysis ofthe Mlanged section isto ‘determine the postion of the neutral ads. If the material properties, Le, Ca Gu and are known, the critica ‘eatral axis can be determined from stess distribution as follows: 4 © Tre ma) ‘The peural axis nay lieder inthe flange orin the web. On the oer band if the crss-secdonal dimensions, ie by Dy Bed and the area of steel A, ae know, the deh (2) of the actual neutral aris can be determined asiollows: Case : Dy The neta axis isin the web, as shown in Fig. 2.11 (6). To compute its depth x, equate moments of vansformed seas of compression and Lension ids about the new ais De (e~ D2) + by (c= D)(e= D2 = m Ae d= 2) @2n However, the compeesion akon by the wes very small as compared lo that taken by the Mange and hence may be neglected bDi(e~ D, (2) = mAg(d ~2) aw ‘The soluon ofthis Eq. (228) wil give the dep x ofthe neural axis, ‘Thus, wile calculating the depth ofthe neta axis, cis essential wo assume fr frst ual tat ether x < yor x> Dy. To reduce the caleattions forthe fest al, assume thatthe neural axis coincides with the undemeath ofthe ange seb and calculate moments M, and M, respectively of tansformed areas of concrete flange and wansfonned area of sce! about the botom of Ue ange, Then “cp Sean eee aoe 46 Reinforced Concrete Deen - i Sauteal { Tet ” ais i Nauteal ie Aye, | ae jis Se] fit & LL as viele (e) Serece airtelewcion i crshing (0) Nevtral axis in flange (6) Neutral axis in wed Fig, 2:11 Normal stress dstrbutlon in renforced concrete T-beam under flexure (0. iM, > M, the neutral axis lies in the Mange, (i) iM. = My, the neural axis coincides wit the botom ofthe Range, and Gil) if, 2 Me neural ani ein the web ‘Lever arm: Depending upon tbe positon othe neil axis there willbe two css of lever ar. Cas : the neal xi ofthe section tls within the ange D), ten the analysis ofthe section is the same as fra recianglar bam, because the elect of concrete below te neural aus has been nelected 2njd=(d~ 30). Case: ner ais ies in te wed (> Dt wt expressive fre conse of te cmpesive fore in the flange and that inthe wed. Te compesive force lathe web being relat) vey small ou 6 negli the analysis The lever ann ofthe secon depeds one poston of tn cea of tn peal suess diagram as lusrated in Fig. 2.1 (). The stance of ie cen of the apezoial sacs Jagan from he tp oft Range given by FalCat2 0, Momo )1(0,!3) 22 where 0X10. ete suesssat he lop and otiom ofthe flange, respectively. From the sess distribution agra: Om Fag l- B)I4] and gem Fan ld=2)12 Tous, F=[Qx~2D)/2x~DI}O,I3) ‘The leveran, 2 (=jd)=d—¥, ‘Moment of resistance: 1 the neusal axis fallin Ue ange the moment of resistance of the section Is My= Cz= b,x (I (d- x3) e320) 230) ean MyaTe=Ay Fuld 28) enn FFor the cae when the neutral axis fas inthe web, the moment of resistance with respect to compression is M,> ODAC +0" Viz where 233) and that with respect to tension is Mabe 30) ‘Asin the case ofthe rectangular section the allowable moment of resistance ofthe flanged section is governed by pernssibe stresses in oth concrete and sec, permissible stress in szel or permissible ses in concrete, ‘depending on whetber the section i balanced, under-einfrced o over-reinfrced, respectively. Working Sess Design: Fleure 7 2.462 Types of Problems Type I: Determinarion of Moment of Resistance [tthe dimensions ofthe section, area of tension stel provided and permissible suresses in the materials are given, the folowing procedure may be adopted: (@ Detennine the depts of the critical neutral axis x, andthe actual neutral axis Ascertain whether the section is balanced, under- oF over-einorced. Ix < 2 the beam i unde-reinforced, and if: > te beam is over-einorced i) Based on the controlling permissible sess compute the sues in he ote mater (Gi) Compute the lever an. (Gu) Determine the moment of resistance. ‘The nature ofthe problem and the procedure for analysis are Musested io the folowing examples. Exaimple 2.10: A reinforced conrete Tee beam section having an effective lange width of 1500 mn, ange thickness of 100 aun, width of web as 250 mm, and effective deph of 450 mun, is reinforced with () _ 22mm & bars, (i) 5%20 mm © bars, on tension side, Caoulate the moment of resistance, and sesses indaced in he exteme compression fibre of concrete and instee. The materials used are MLS grade concrete and sce of grade Fet5, Determine the uniformly distributed load inclusive of self-weigh the beam can support safely over an fective span of 65m, Solution. For MAS concrete and FeA1S ste! reinforcement: Gee= 5 Ninn, oy = 230 Nim? and m= 19. ‘The sectional properties ae = 1500 mm,d= 450mm, D,= 100 mm, = 250 mv, and Ae =622 mun © = 280 mi, ‘The position ofthe eritcal neural axis is obtained fom kd = 029 x 450 = 130.5 mam (k= 0.29 from Table 2.1) (Case 1: To ascertain the position ofthe actual neutral axis, ake moments M, and My respectively, of the transformed area of coneretein he flange and the vansformed are of sel about te btton ofthe flange. KE -M, > M, the neutral axis falls in the ange, and if M.< M, dhe veutal ais lies in the web, For we present problem 1M, = BD, (D112 = 1300 1005075 x 1 ma, and M, = mA, (d~ D)=19 2280x350 = 152 10 we! Since M,> Mth nota ais lis in the Web (> D).Equate mements aout the neural axis: 2D,@-D,I2=mA,(d—2) or 15003 100% (= $0) = 19 «280% (450 ~2) ving x= 159.63 um Since > te beam is over sinforool and hence y= ge #5 Na. The coresponding Sees at be boom ofthe ange, ad in tension ste ae: a= Fa (k= D) = 5x (13965 ~ 100)7 13963 142 Ni? Sgn md qd 2112-195 (450 ~ 13863) / 13968 = 211.17 Nin, “ ‘The distance of the enti ofthe compressive fre frm the op ofthe ange: al Seet 20m) .Ds_ (5423142) 100 - (3)4 (8 12) = ‘Averatvely 7 = Gx~ 2.0) B, (3x ~ D)] = 40.70 mm, Thus the lever an, 2 #450 = 40.71 = 409.29 171 en. 48 Relyjorced Concrete Design Mp2 C22 [b Diag 0a) 212 = 1500 100 x (5 + 1.42) x 400.2972 = 197,073,135 Nam = 197.07 Na, Let ete nifry dtd oa ice of e-velg nL ebm an yy, apt ‘maximum moment M to M, a Case ll: Ay=5%20mm = 150mm, f Momens of wansformed areas about te botom ofthe flange ‘Mz 1500 «100% 50= 7.5 10° mum? M,= 19% 1570x350 10.46% 10% mun’ Since M,> Ma the neutral axis falls ithe web (x > D). Thus b,Di(e~D,!2) =m Ay d=3) 1500 x 1006 5 siving x= 11635 mm, Since x< 1. the setion is underreinorced wih = 0,» 250 Nin’, The coesponding sresses in ‘concrete atthe top and botiom ofthe Ange ae: ge tk 230x116: =n =a” Dx ASO= 11635) 9 1570 «(450 ~ 2) = 422Nhon? (=D) _230%(116.35~100) im (dma) 19%(650=116.35) Depth ofthe centsid of compressive fom the top ofthe ange: Fal a+ 20%) (Cae # Oa) X(D,/ 3) = 3742 mm, Altematively, Fe (Gx-2D)/Qr- DI} xO,/3) = [6 «116.35 ~ 2 100) 12x 116.35 ~ 100) x(100/3) = 374mm, Thus the Lever am, 2 = 450~ 37.42 = 41258 mm, Allowable moment of resistance, M, = Tz: M,= (Aa 0) x= (1570x230) 412,58 148,982,638 Nmun = 148,98 EN, Letirbe te safe distributed load inchsive of self weight in kN, Equate maximum moment Mo allowable ‘ent of esse Mz WX (6.5778 = 148.98 Therefore, w= 28.21 ENim (say 28 KNAm, 059K, Type i: Checking the Adequacy ofthe Section ‘The cross-sectional dimensions b, Dy. Pu d, and aea of tension reinforcement sve given It is required to ‘compute the stresses developed inthe materials under the action of given loading The steps involved ae: (i) Determining the depth ofthe actual neutral als as explained in Type | problems. Gi) Computing the ever a, Working Svess Design: Flesare 49 (i). Determining Min terms of known tensile sues a equating itt he moment duct extemal toad, and determining Calculating using the stress diagram, Example 2:11 'A singly reinforced concrete beam of T-shaped section has an effective lange of width 1500 mum and thickness of 125 mm. The tension reinforcement i provided alan effective depth of 600 mun, The bean suppor uniformly distributed load of 45 EN Gaclusve of slf-weigh) over effective span of 6m. Te grade of concrete mix used is M20. Detemne the sesses iced inthe top compression fibre ‘of cccrec and in tension tel forthe following cases of tension sie: () 620 mun @ ad (i) 6 28 an 6 Solation. The propenies of the T:secion are: by 1500 mmm, D,= 125 mn and d= 600 run. For 20 grade concrete: Ou ® 7 New m= 13, Maximum moment, M = wET8 = 45 x(6.5)'8 = 237.65 kN. Case I: Ay = 620mm 9 = 1885 rn? Moments of transformed areas about the boom ofthe ange: ‘Mex 18003128 (12872) = 1.725108 ma? ‘My 13 x 1885 x (600 ~ 125) = 11.64 x 10" eum’ ‘Since M,> M, the neural axis es in the ange. Take moment ahoct the neural axis: bta2= mA d~2) Pemers- 19604 = 12463 man D), Take moment bout the neutral axis: BD Ae~D,2)~mAc(d~2) 1500. 125 (= 625) = 132944 (0 ~ 2) tiving = 15361 mm, The depth ofthe centri from the tp of he ange Fe (Gr-2D)D/A30eD)) =O x 15361 = 2129) « 12519 x(@x 15361 ~ 125) 4821 mn, Fx 600~4821 =581.79 mn, (237.66 10792944 x 551.79 es | Mé20x15361 Fe G=n 3.87Nia. 4.553 ‘Suess inconcrete, 50 Reinforced Concrete Design ‘Altematively let a, be te sess at the extreme compresion fibre; then sexs atthe botlom fibre ofthe ange: Ong = Oa Dis = Ogg (153.61 ~ 125V 153.61 = 01860 ‘Total compressive force in ange: C= og, (1 + 0.186) x 1500 x 12572 = 74.125. “Therefore, M, = C= 74.125 6/4, x581.1. quate M, to Mt: Fg =(237.66 x 10°M(74.125 x 551.79) = 387 Nowa, Corresponding stress in tension steel: 2 aed shb= 3.87 x (600 153,619/153.61 = 146.20 Ni Example 2.12: The floor of a rectangular hall 6 m wide tothe centre of supports consists ofa 100 mam {hick slab cast monolithially with and connected to rectangular beans spaced 3 m cente-o-celre, as shown, in Pig. 212. The with ofthe web and effective dei ofthe beam ase 250 uum and $50 mm, especivey. ‘The beams are reinforced with 6x 25 mim @ mild sel bars ou the tension side. MIS grade concrete is used in construction. Analyse the end beam for: () determination of allowable moment of resistance ad (i) the ‘stresses inced inthe top compression fibre of concrete ad tension steel, if ube beam suppors a niformly ‘isributed load of 40 kNim Gnclusive of self-weighd. ‘Solin. For MIS grade concrete and mild see reinforcement: ae = 5 Ninun?, cq 140 Niu and m = 28013 > 5) = 18.65 (= 19s stipulate inthe code). ‘The cross-sectional properties of the end beam which bebaves as an L-beam ae: y= 100mm, b, = 250 mm, d = $50 mm, A, 6% 25 mm $= 2945 mn", and the effective wid ofthe ange is given by b= (ID) +b, + 3 Dy= (6x 1OQO/I2) +250 + (x 100) = 1050 mm, Position of the Neutral Axis ‘Moments of transformed areas shout the bouom ofthe flange: ‘M, = 1050 100 x (1002) = 5.25 10F ma? 2M, = 19 x 2945 x (550 ~ 100) = 25.18 x 10m’. Since M,< Mj, tue neutral axis les im he web, ie, > D,. Take moment of tanstormod areas about the ‘neu axis! Df ~ Df m Ay (d~ 2) 1050 x 100% («~ 50) = 19 «2945 (550-2) . siving x= 223.82 w To ascertain whether the section is balanced, under-vinforced or over-reinforced comput the depth of| the excl neural axis, x2 ondill + (fm 04) = S501 + (140/19 x 5)} = 222:34 mun, Since.x> A, the section is tobe analysed as an over-reinfored section. Hence og. dg = $ Noun and the stress atthe bottown ofthe flange Og, = Oa. ~ DV = 2.767 Ninun?. ‘The depth ofthe centroid of the compressive force from the top ofthe flange: Working Svess Design: Flere St 6.0 0 Spandrel or a "thames so2.250rh ho. rah _ (b) 1080 aw} Perea 250 wn lg. 212 Design oa relafored covcrete T Scam section $2. Reinforced Concrete Design (Sa) 22g pe am =D, )°3 "ama * 3 =4521 mm “hetfore ever arm, zm d— ¥= $50 = 48.21» 50479 mm, “hos, momeat of resistance wih spect compere foe: My= DAC ue OMIM 2 1050 «100 (542.767) 50479 = 205437 nm = 205.57 Nn The beam suppons a unonny dsb oad of KN, The maximum manent Ms given by M= m= 40x68 18088. ‘Suress in steel, of = MIA, (180 x 10°)/(2945 x $04.79) ‘Stress in concrete, 0. = (g/m) alld — 4) 121.0918) 22882550 ~ 22382) 4309 Nn? eat, et ab te resin concreettc enee ibe ote Mange Then isin We conc ste wider ote flange sa O'S = De= 05530 Moment of sacs wid spect wo compres oe: M =H DU + a2} X2= AIST HO une 10 415710 a = 190% 10, vi 121.08 Ninn’ 373 New Type il: Design ofthe Flanged Beam ‘The design of a section requires the determination of cross-sectional dimensions and the are of stel for resisting the applied moment The thickness ofthe Nange Is xed bythe design ofthe lab and te effective ‘wid ofthe Manges thapatof the slab which deflets monaihically withthe beam andis under compression. ‘is governod bythe ertera given in IS:456-1978. The widh of the web is based on shear and reinforcement placement considerations. Thus the design of the Tee r El-beam section requires the detemminaion of te effective depth of We beam and the area of reinforcement. The seps Involved are (0 Estimating the deh ofthe langed beam for preliminary computations as follows: (@) 1/124 ofthe span for heavy toads, (@) 172 to VISth of the span for medium lads, and () 15d w 12044 fo ight Toads. Gi) Estimating the ib wid, whichis approximately one-half ofthe rib projection below te slab ange (Gi). Determining the maximus. bending moment M 10 be carid bythe beam, G0) Checking the dept from econoeni considerations: d= 2)41M MOP ay where ris cost rai of set to concrete. (0) Ascertaining te type of section, i, whether tis balanced, under reinforced or over-tenforced, GD Caleuatng the sectional area of reinforcement tased on oF on assumed approximate value 2= 0:94 or @-D,22) (ii) Based ont sectional area of reinforcement cbained instep (i), determin the neutral axis aed hence Wie wut Value ofthe lever am, i he actual position of Working Sess Design: Fesure 53 (i) Recalculatng the sectional area of eeinforcement. (2) Checking he resulting maximum streses in concrete and tel, Example 2.13: A reaforeed concrete flor sysem consists of 100 mim thick sib cast monolithically with simply supported bears havin an effective span of 65 m a spaced 3 m cenré-to-cene. The super- ‘imposed load on the Noor s 6 KN/n’, Design atypical intermediate bea, ifthe materials be used se MIS rade concrete and mild steel reinforcement. The unit weigh of concrete is 25 ENA Solution. For MIS concrete and mild tet reinforcement Sac 5 Nin’, 0, = 140 Naan’ m= 19, and k= 0404, Consider total depth of approximately 1/12h ofthe span, i. say $50 mun and the width ofthe ib a5 = (550 — 100V2= 225 nan, say 250 sa, The effective wid of the ange by isthe minimum of the following: (© WYO+b, +6 Dy= (650016) + 250+ (6x 100) = 1933 mm (3m, Le, 3000 mun ‘Adopt ange wiih, = 1933 mm, Loads on Beane ‘Self-weigh of stab, = 0.10 x25 254Nimt Superimposed load 6OKNin* ‘Total Lad on slab = 85KNm ‘Land per metre length of beam = 255kNin Selfowcight ofbeam =025x(055~ 0.1) x25 = 28125KNin “otal uniformly distributed Yaad onthe beam, w= 78.3125 Nin Maximum bending moment= wf = 28.3125 x 6.518 = 149.93 KN, Bconomic dpi, d = (D1 2) + (Mr 6.) Assume cost ratio, r= 60, ten (1002) + 1049.33 x 10F x 60/140 x 250)! “Thus adopt a otal depth of $50 mm with d = $00 mam, ‘Assume the section wo be balanced: = 0404 d = 202 mm > De{ = 100 mu). ‘ ‘ence: the critical neutral ais falls in the web, and the stress underveath the lange: n= Tal ~DIlt, = 0.505 Fe ‘The éepth ofthe resultant compressive fore in concrete from the top ofthe ange is given by: F, #1Gx,-2D)2x,- D)].Of)= 4452 mm, Leverann,2, = (4) = 500 ~ 4452 455.48 mm, For the balanced moment of resistance: Cae $Oq= SNe? May = 05X06 5,4 64) 5,012 50.55 x(1+0.505) 1933 x 100 455.48, 331,266,619 Naan: 86.3 ram, Ko, 2.) = 149.53 x 1O5M(140 » 455.48) = 2348 ‘To determine the postion of he actual neutral axis, tke moment of wansfonned agvas abou Ui neural ‘S€ Reinforced Conerte Design b,D,~Dy/2) 1933 x 100 (x 50) siving x= 134.29 mun and Z Lever arm, z= 459,89 mm. ‘The exact area of tension see: Ags (149.53 x 109/140 45989) =2322 ma ‘The stresses i the materials are: = 140 Nin’, and from the stress disuibution diagram Fax™ (140) (134.29) 1 (500 ~ 13429) =2.71 Nina, Mm Ag d~3) 192345 x (500-2) [Gr-2D))/Ax~ D)O,2)=40.11 ma, 2.6.3 Flanged Reams with Teasion and Compression Reinforcements ‘When a lange section is reinforced on tbe compression sde also itis known as a doubly reinforced flanged ‘section, Icean be designed by using aprocedare similar to hat fora doubly reinforced rectangular beam, The ‘eutra axis may le either inthe flange or in the web. To ascertain this conser tht tbe neal axis coincides ‘th he undemeath of the sab ange, and compute moments M, and M, of transformed areas on coxnpession snd tension sides, respectively, about the botlom ofthe flange: M,= BD, (Dyi2)+(1.5m = 1)A,{D,=2") 35) M.=mAJd-D) 239 11M, > My the neatral axis is inthe flange, On the other hand, if M, D), the section is analysed asa T-beam section, aed {We depth of the neural axis can be computed from: bp Dx ~ DP) +(1.Sm~ DA) = Ald 2) 238) ‘Tae lever arm depends on the postion ofthe resultant compressive force. The stresses at various levels in ters of the stress induced in the extreme compression ibe of concrete) can be obtained from the Suess Aisributon diagram San, Su Smo d=) oa Ts 7 Og" Fale Er: g(t Dis: tnd = m0 ld 2k 0) Since te compressive force in the web is elatively very sali comparison with he compressive free the lange ity be neglected in the analyis ‘The postion of the centroid of connpressve force in concrete C from the tp ofthe lange i given by: Fe= (ut 20%)! a+ On) (0,13) ‘The sane ofthe force in compression sel C om te tp ofthe Manges. Tous Fa (CE+CaV(C +c) ean Tre lever am z(=je)=d-3. ‘The moment of ressince of he section is given by: (1) Wi respec wo the eampeessve force M,=lOS +0" My Dy+(1.Sn DAS Ie M,= 0.509 40g Dy ld ~ (2) With espect ta te tei force 415m ~ Ao ld ~d) 2a Working Stress Design: Flesare S$ Ma Cbd: ea ‘Teena the problems an proce or sali eluate inthe folowing examples Example 2.14 The section oa T-beam shown in Fig. 2.13 basa lang of elective wand athichness 300 mm sn 125mm, respecvely. Te eect dephf te beam and te wid of te wear OD tt 40 300, respectively The eam is enforced wi oa as Of 25 mm on te tno ide nd ee bers 022m 4 he cnpesin side The effective coverto compression scl 90 aun The mara usedave M20 ga cone and HYSD stl of grade Fo415, Deen: (allowable moment exits, (i tbe usifomly iste lad the beam can Supp sully over an eect simply supped span of & 3, and (i) sees idol inthe top compression fb of ene and tension scl when the bean i subjected oa ending moment of 2001N. 4h A300 am be=1500 em DpnA25 nm 4>600 mm 4°50 wm Age 425 ame Agen3-22 am @ (2) Cross-section (b) Forces on the section ig 213 Anayse of doubly relforond Thonn section ‘Solution. For M20 concree and Fed1 stet: ‘Ninn’, og = 230 Nun’, and m= 13 500 mam, D,= 125 mm, b, = 300:mm, d = 600 mm and d= 50 mm ‘To ascertain the position ofthe neutral ais, take moments of transformed areas on compression and tension sides about the bonom of he Mange: Myx b,DiD,I2) + USm~ Ag (Dy 2) = 1300 125 x 62.5 + (.5 x 13 ~ 1) 1140 x (125 ~ 50) = 13.30 10¢ mun? Mex mg (d~ D) = 13x 1963 «(600 ~ 125) = 12.12% 10 mar, Since M, > M, the neural axis les in the ange. The exact position ofthe neutral axis ean be dtermined by ‘aking moments of vansformed areas about the neutral ais bxGID +S m~DAG- Cem Ayd=2 1500 272 + (1.5 «13 ~ 1)% 1140x (x- $0) = 13 1963 x (600-3) P-O1St-219212=0 [6 Relgorced Concrete Design siving x= 119.88 mm (< D9, ‘The positon ofthe balanced neutral axis given by 2dl! + (6Jmoq)}= GOO + 230/13 x7)]= 170.09 mm, Since. x, the section is under-einforced and hence = 0q= 230 Nam’, The siresesin theextreme compression fibre of concrete and in compression sce canbe obained as 0,119.88, zi me d-x 7 1360-11988 Niet (LS gt dV Sx 13 x4.42x(119,88- SOVI19.8 = 5022? ‘To determine the allowable moment of resistance ofthe section, tke moment bout tension see: M,= £05 Ou) d= 23) + Ae d=) += 1500 119.88 «(05 4.42) x (600 ~ 119.887) + 1140 «50.22 x (600 50) . = 254.05 x 40" Nan = 254,05 kN, (Gi) Let Nin be the total uniformly distributed load (nclusve of sel-weigh the beam can support salely, The maximum bending momest, M=Wx88= 8 w EN Equaie M to Mf; 8 w= 25405, giving w= 31.76 Nim. (ii) Since the beam is mnder-reinforced, equate allowable moment of resistance with respect to tensile force otha wits respect to compressive force obwined earn: My Ay = 254.05 x 10" ‘Therefore, lever arm z= (254.05 > 10°V(230 > 1963) = $62.69 oun, ‘The stresses in the materials duet a moment of 200 KN: fa (200 x 10°Y(1963 x 562.69) = 181,07 Nin. From the stress distribution diagram: Fae (fm) x/(d~ 2) = (181.0713) 119 88600 — 119.88) = 3.478 Nina? Sm(odm)(e- avd ~2) = (15% 181.07) (119.88 ~ $0) (600 ~ 119.88) = 39.53 Nima? Avtematively, the szesses induced in materials due wo applied moment can also be computed by multiplying the suresses obtained in pat (by We rao (MIM) ge 4.42 x 20025405) = 3.48 Nun? of, =90.22 x(200254.08) = 39.54 Nim? fg = 230% (200254,05) = 181.07 Nm Example 215: A typical beam ofa floor system behaves a8 a T-beam section having a ange of size 1600 x 125 mum, The width ofthe web adopted is 300 mun, Design the T-beam i itis subjected toa moment (0 300 kN m for the following cass: (there i no resriction on ube depth ofthe Beam abe (i) the effective ‘depth of team is tobe SOO mun. Use M20 grade concrete and HYSD ste! of grade Fed as reinforcement. Soluuon. For M20 grade concrete and Feds grade sek: ac = TNmm',¢, = 280 Nina? and m= 13. A, = MM #230003 x7) = 0.2835 and j= 1 £,/3=0906, x £O283S¢. Since Mya > M, Working Stress Design: Flare ST ‘The known eros-sectional dimensions ae: = 1600 mm, Dy= 125mm, and b, = 300 mm, Case I: The besa may be designed asa balanced section vit the ntl axis alin inthe flange. For this case, the section wil bchave a8 a rectangular section with b= by and May =O Ga bx (4X13) 5 x7 x 1600 x0.2835 dx (1 -02835/9.d= 1437.572¢° Num, quate My tothe applied moment M: 1437572 d= 300. 10f, giving d = 456,82 mun, For the neutral axis to lie in the Mange 5,< D, Le, 02835 d 125 or d< 440,92 ma ‘Adoptd= 435m; the nevtral axis wil iin the Mange. The allowable moment of resistance of the singly ‘enforced rectangular balanced section: Muy 1437.572 x (435) x 10 = 272.02 AN Since M> My itis tobe analysed as a doubly reinforced beam, The area of steel Ay, for resisting Mate given by: Aa = Ma Hy) = (27.02 1057230 0,006 » 435) ~ 3000.93 mun? Additional moment (M ~ Mi) is resisted by additional tension steel Ay; and compression ste Ay. Let the effective cover to compression tel, = 35 mn. Then ‘gq ® (300 ~ 272.02) « 1071230 x 435 ~ 35) © 304.13 mn. ‘Total area of tension see, Ay = Aq + Ass = 3305.06 mm ‘Tuc compression sie! may be obained by taking moment of compressive force in compression steel bout the centroid of tension steel with : = 0.2835 435 = 128.32 mm, Ma Myo (1S m= WA Gael 21x) dd) = (15x13 ~ 1)A.x7x [023.32 ~35)/ 123.32] (35-35) 3709841 4,. Therefore, Ay = (900 ~ 272.02) x 177,098.41 = 7542 mm? Cate 1: The effective depth inthis case is 500 mun. The design ofthe section requires the determination of tie area of reinforcement. To ascerain whether the section isto be singly reinforced, balanced, under: ‘reinforce or doubly reinforced, assume the section wo be the balanced one: #02835 x 500 = 141.75 mm>D,. ence the critical neutral axis fallin the web. The compressive stress a the underneath ofthe flange sla: Oar Cac GDM, Fg, (I4LTS ~ 125)141.75 2 0.1182 6 Depth ofthe resultant compressive force in concrete from the top of the Mange: (met 20" a Moa, + 0'u)}X(D,/ 3) = 46.07 me, ‘Adteruatively, = (( x, ~ 2D Mx, ~ D)} x (D3) = 46.07 mm, Lever arm, z= d~¥,=500~ 46.07 = 453.93 mm, For the moment of resistance as a balanced section, oq, = 7 Nim’: Maa= Dil Oa) 1 1600 > 125 7 (1 ¢0.1182V2] x 453.93 = 355,300,168 Naum is an under-einforced section, Tus, Fg 04 230 Ninn, ‘To ascertain the postion ofthe new ans, et it coincide with the bonom ofthe Mage, ie, Dy and Og = (230/13) (1250500 ~ 125) = $897 New? [58 Reinforced Concrete Design ‘The moment of resistance fortis case is My = 05 0/4 bsDy(d~D,13) 5 >¢5.897 x 1600 x 125 x (500 ~ 125) x 10*= 270.28 kN, Since M, 415 Nin?) ‘The characerstie srengths for various types of steel ae given in Table 34 ‘Table 34 Character Strengths fr Various Grades of Stet Types ed ade fal [Characters aength fy] Perineal ‘Nia Ninn Wid wel pe 70 140 Medium ten defomed bars io 180 HYSD set of grade Fess as Bo YSD fel of grade FeSO0 as hs 32.5 Maiimam Spacing «+ Theshear reinforcement is provided to prevent the shear cracks. Its seen hat the horizontal distance betwen two sucessive cracks is approximately equal othe effective dps dA stirup shal be provided such that it ‘crosses the potential crack and also no potelal cack shal ain uncrossed To ensure ths, the maxima Spacig of shear reinforcement measured slong the axis ofthe member shall not exceed 0.75d for vertical $Brrpe and dfornctned sop. Inno cae stall be spacing exceed 450 min, 3.3 DESIGN PROCEDURE FOR SHEAR REINFORCEMENT ‘The cross seconal dimensions area of Nexure sel, properties of materials used, and desig shear force are ‘given, The steps involved in the shear design are: (@ Devermining the maximum shear force andthe coresponding nominal shear test Computing steel rao (100 Ab) and fendi th coresponding shear senguh of concrete. Gi) Comparing «with, (0) I,< no shear design is equied, Provide nominal vertical straps thoughout the beam where A, is the tual eros sectional area of stip ees. - VIS,

60) = 1.25 Nim. Since €, << Tum the Seton is adequate wih design shea enforcement. Design shear free Ve=V— 1 bd = 725,000 — 0.376 x 300 x 600 = 157,320 N, {Lettwo20/mn bars Ay = 628 ra’) be ent at 45°, Shear resisted by the bent up bars = Oy Ay Sit c= 230 628 X in 45°» 102,134 5 However, shear provided by beat up bares limited wo V2 (= 78,660 N). ‘Thus hear foot for the design o stops: Vg= 157,320 ~ 13.660 = 78,660 N. Use 8 mm ¢tworkeged vertical mild ste trups (Ay = 100 man: ‘SG A dV = 140 x 100 x 600(78,660) = 10679 me. 751V/m 200 ay Pe Tn As ie nn ¢ 10.376 MPa 1.25 MPa Cross-section 1.25 MPa. * ban wa (a) Variation of shear stress oes oon ofison ae (b) Spacing of 8 mm ¢ 2-legged stirrups Fig 3.4 Shear renforooment tie for the snply supported beam of Example 3.4 10 Relyorced Concrete Design ‘The spaciog shal be he least of the folowing: (9 078x600» 4s0mm, Gi) 40am, 1 (8), 20839 mam (minimum soar etaercemen criterion), and @) 10679 mm, ‘Toe shear ses istibtion in te eam i sbown in Fi, 3.4) Lette distance measured from the face of he support where shear sues eu: 0.376 Nin tet 0316125=0- 23 Hence, x= 2.0976". Pace the fst stinyp ata distance 0 man (= 5, 2) from the saps and thee provide 8mm 4, legged mild tel vertical serps @ 106 mm cle wp to 2.1m fom te support Forte remaining pion lncrease te spacing to 208mm ef, ‘The ars ar bent vp a 1.25D rom te suppor whore D = 600+ SO (sumed cove) = 680 mm, ie. a 8125 mm (ay at 800 mm. Example 32:. The enforced concrat canlever eam of pan 35m, shown in Fig. 3.5, bs secon 300m wide au 50 wm dep (effective) ate Iae Of We soppor. The Sp Ts edoeed vif 0280 3mm effective) alte fe nd The beam rence tees face wit bars of 20h a he sepron Twoof these tnsionbars ae curled ata distance Of125m fromthe suppor, Design sher enforcement! for the beam when i caries a uniformly Gbuted erie kad of 6D tN, Te mates wed are MO sae emerete and HYSD ste of grade et Solution. For 20 rade concrete Cn = 1.8 Ne’ ‘Aube support: b= 300 mm, d= 550 mun: ‘44 (6%20 mm )= 1885 mn? ‘ 7 = 100 Ag ie = 100 x 18851000 550) = 1.142 1% 039+ (042 ~ 0.39) (1.142 ~ 1.000(4.25- 10) Maximum shea force andmoment at the oppo Ve wl= 6033 $=2104N Mav= GOxG5)2= 36158. [Nominal shear sess 1% ={¥s Quid wn Bed) here tm = nent of slope of boom ce = (600 ~ 30093500 = 0.0857 (cover asumed as $0 mm) ‘The ~ve sgn wil be sed asthe dep increas wid he nreasenRending monet. Ths 5 = 110% 10) ~ (367.5 x 107550) x 0.0857)(300% 550) = 0.926 Nin Since << Tuy the section is accepable wit scar reinforcement. The curtailed longi bars sxc effective in resisting shear up othe poi B which at distanced from te pint of eutament Bs sown in Fig. 3. (eT shear strength of concrete depend on te agit son sel Ths, for he length A shea such of courte, = 040 Naw effective ph at point 8 = 20 (3003500) 2250) = 442.86 mam fective depth at pin Bd = 250+ (300/350) x (250 + 42:6) = 48082 mn Atelfetve ett point Ay 420 man) = 1256 m= (P= 100.4 a= (100 > 1256300 x 40.82) = 0871 407 New? Shear:Bond and Torsion 1. 300 m fFF51— 6-20 um ¢ TE ale ol Section at support i 3.50 (@) Cantilever bean Fi 0 KN/m ct ee (t) Shear force difgeen BYOB s+ (©) Outoff point ra 1.5m [Stomp 2690 mm [mi ig-35 Shear reatorecment tals forthe canteerof Example 3:2 ‘Shear strength of concrete: 20.35 + (039 ~ 035(0H71 ~0.75)41.00 ~ 0.75) = 0.369 Nina’ Nominal shear sess, = [V ~ (MAf tan BIXBd) where Vax (2250-40443) = 161.572N M=60x0.250 + 0443/12 = 2745 Nm 72 Reinforced Coneree Design “Therefore, ‘= 1.572 x 107 ~ 217.545 x 171480 82) x 0.0857}/300 x 480.82) = 0.8513 Nin? Sing > fae efrcment equi, Ur 8 mn 9p vera HYSD Sel straps (A= 100 Spacing in length AB $= (Gy Ant, ~ ¥)b] =230 x 100)(0.926 — 0.407) x 300] = 147.72 ran, “Maximum spacing tase on minimum shear enforcement: 5,5 An fA B) = 100 415104 x 300) = 345.83 mm. Spacing shal be the least ofthe following © 075 d=075 x480.82 = 360.6 mm, (i) 450 oun, i) 345.89 nm and (iv) 147.72: eid. Provide 8 mm @wo-legged sisrups approximately 47 mm cl cay MS mu cs), ‘Spacing of serps inthe length BC: ‘= (230 1009/0 8513 - 0.369) x 300] = 158.96 man ‘Spacing shall be te least of the folowing: © 095d=0.75x250= 187.5 mm, (i) 450.0, Gi) 345.83 mun (iiznum sted, (2) 158.96 mm (required) ence provide 8 mm @ two-legged stsraps spproximsicly 158 nim cle (sy 155 mm ck. “The shear reinforcement is shown in Fig. 35 34 BOND AND DEVELOPMENT LENGTH. For reinforced concrete 1 behave effectively, itis essen that suesss be transfered from one material 0 We ober. In other words, tere shouldbe a slippage o¢ relive movetnent between conrete ad the rein. forcement. The grip of concrete on renfercement eto adhesion or bearing is tenmed the bond Wrest any {oee eng to case slippage ofthe bar relative to the surrounding concrete. The resulting shea stress st the imerface of the bar and concrete is known as bond stress ands defined a force per wot ominal surface sara of reinforcing bar. The bond between concrete and see! should be perfect at sevice loads since Tht validity of reinforced concrete theory is hase on te ffiment ofthis condition ‘The bond sess developed is du to pore adesion, fictional esisance de o eripping af reinforcement by conerete because of shrinkage, ead the mechanical resktance provided by deformation o twisting oa (he surface of he bar. Failure in bod in concrete wid plain bars occurs when adhesion and rctonalesisance ‘ae overcome and the bar is pulled leaving a round hole. To prevent this, end anchorage is provided mostly Inte form of hooks. On the ober band, a deformed ar increases the bod capacity duc to mechanical resistance ‘inaddidon to adhesion ad fictional resistance. When adequate emedment length is provided, bond falar {ue paling of bar des not occur nd flare wil be normally by sling of concrete. According to 1S:456-1978, the calculated tension a compression in aay ba a ay cros-tecton shall be develope (by flexural bond) ou eacn side of the tar by appropriate Bar length or ead anchorage of & combination ofboth. The extended lng of the bar equied to tans the bas force wo concrete know 233 development length L,I is obaived by equating Ue bond resistance of concrete to the sength of de ‘einfocing bay, as asrted in Fig. 3.6: Shear, Bond and Torsion 73 A. 4 Tha 1 + Least Lo ve outa tutte “ Ler o4 4) Gal) whee 4, = development eth “Ge = permissible average stein bond 4 = nominal diameter ofthe bar ©, = stress in reinforcing bar atthe section under consideration athe design Kd ‘The permisible average bond suresss for plan and deformed bars in tension and compression, a c= ‘ommended by 15:456-1978. ae given in Table 3.5, be 35 Perms Stresses a Bod for Diferent Grades of Concrete . Coe pe BY Tae tev_| ow | os | ovo | roo | so | 120 Temion Dito tar | ose fuse | ras | ao | ise | uae Te Nima? Pain tes] ons | too | vs | ras | iss | 150 compesion | Drom ter | tos | sao | isos | tas | iss | 210 Nowe: (for compression brs is 25 pe cet geste tan al fo oson br 2. For defonmed bar the ales yar lcreasd by 40 pce. For the portions ofthe member where bending mament i nearly zero, vz. nea he supports of simply supporid beams and nea the pots ofiafiexion the exteraa shear orc may be large andthe are of ese ‘einforcment may be small thus making feturalsueagt cra Considr two secionso bean a disc: dc apa, he incremental force dT produced in the ars de to change in momcat by dM is given by dra dM iid. ++ TE Eo isthe periphery ofthe bars, then for euiibrium of kngth dof bars, SydtEo maT =dMijd dM _dMids Vv Geis “ae @.2) 1 Relfored Conerete Design ‘The development length to develop the strength of he bars canbe obtained from the relation: A= bho oy Substnwe the value of from Bg. (3.12) imo Eg. (2.13): Au= WHAEOHL LO = (VEE whence Le ASO =I o19 ‘where Mf; is moment of resistance ofthe section assuming al tension reinforcement to be stressed 10 Oy If Ieaverage bond strength isnot tobe exceeded, the ratio (MV) oust be ual oo ager ian the required development length Ly. To preclude the possibilty of bond suength being exceeded even locally, additonal anchorage length Ls provided. Ata simple suppent, Lis the anchorage length beyond the cone of support ‘and he equivalent anchorage vale f any ok or mechanical ancorage. Atapoin of inflection, Ls imied (ode effective depth ofthe member o 12 4, whichever is greater. Equaton (3.14) is therefore indied 2 LS M/V)4L, G.lsa) In cae the ends of reinforcement are confined by compressive reaction, the valve of (Mi/¥) i to be ‘nrease by 30 percent te 1,8 1304 )V)+ Ly G.156) 35 ANCHORING THE REINFORCEMENT Intenso Deter ay ee wide provi ie evcpmentent esenent ite Hooks soul nooaly be rodeo pan tain aston “he anchorage vale of he en sal be keno be (= Smet of et fr ach 4", subject to asim of 166, The anche vale of he ed Uaype hock tal Regalo 14 In conpeson: The anchorage eg oa sii tai compeson al be ale devchpoest Ico of an compresion The poten tbs, ees and srg gts bond ends ipod for bars compression shal b conde for Soxcopoeat cag In sear: a caso ie bar deel gh for ta nthe tention Zones eased rom te tnd te sping ono porden of te ar, wha he cmresn sas care one 5 dp tte foam Incase ofstraprinte beam sndronnee ican column compe devoting an ache salle dered have ben provid wes bs et ogh an ng ar est 90 unl acteb fate ison dancer and congo lft are fre ap o ath cess (50) erwhen the berben rot arholeo 13" sd eta yond ea ean one fateat 6 wea te basen Oop an angle o 1 andi cue yond te ote focaleng atiea4 9. The aone oven smd ni. 37 36 CURTAILMENT OF TENSION REINFORCEMENT [kiszcommon practic to cutoff bars where they ar no longer equied to sesit moment. nse of continous ‘beams, however, the tension reinforcement a the Botom may be bent up 1 provide som of Ue tension ‘inforcement required onthe top face ofthe bean. For caralment, the enforcement is extended youd the point at which is no longer required to resist ‘enure for a distance (d) equal tbe effective deh of he meme of 12 tines the diameter ofthe bar (12 whichever is greater except aa simple supporter caver, as ius in Fig, 18), In aditon the Tellowjog shoukd- tsa be satite: 4 49min. Shear, Bond and Torsion 75 ua 90° Bend 135° Bend ys se ih Cir=é¢ nin = 4¢min 180° Hook DYE UC bo (2) 15:2502 hooks and bends for stirrups k #2 for mild steel bers and k€4 for HYSD steel bars (>) 15:2502-1963 hooks and bends for reinforcement bars Fig 39 e)Anchorage shear reaforemneats (6 standard hooks and bends "16 Reiforced Concrete Design (9 Tee sexual es, when termina inte tension zone siryp aca in excess ofthat equied for ‘Sear ad trson is povided along cach terminated br over a distance from te cuff point equal to tree fou the effective dep ofthe member. The stip area shall not be less than 0.4 Sf whee bs the wi ofthe beam, 5 is be space a ji teehee suengh of eifereaeat ia Nia, The ‘esuling spacing sal ascexcoed 8, whet ys the rao ofthe Cutt tal aa of bars at the section. (i) In case of positive moment reifecemen, tas onetird of itn a simple member and one-fourth {in acstinvous member shal extol along the sane fare of be member by, beyod he roretcal cute iat of cured bar into tbe suport 1 eg equal 1 Ly 3. In cae a rane member postive ren forcemeat extended ito he suppor sal be ancered to develop design Ses i tension a te fac of the suppor (Gi) At simple soppors and atthe ois of infection, postive moment (ension se tall be limited to gamete such hat development length L(= 64/4) docs nt exceed (MV) +L whee Mite moment Sf resisance ofthe secon asuring al enforcement a the econ wo be seal to Gy [she sum of fnchorge beyond te ene of suppost sn the equivalent anchorage valve of any hook mechanical anchorage alte simple support and ae pot of eso, ys anil othe effective depth oe member OF 12 ¢ whichever is greater. The value of My/V may be inreaed by 30 per cent when the ends Ofte ‘infortement are confined by a conpresve reaction. (i) Incase of negave momen reiafocement, at eas one-hird of he al enforcement provide for scyave meat ate suppor shallextend beyond pon ofinfeton wa distance not ess an dor 124 ‘roe sient ofthe clea spa, whichever i grees For negative moment enfrcment, the brs must {tend afl development engi beyond te fc of the suppor. In aden, he curled brs shal exend bya tance d oc 12g, whichever is greer, beyond the Beoreal cul point of te negative bending ‘moment Gaga. The remaining brs at east oe id ofthe toa) must xtcad a Kast, beyond the tGeorecal eo pot of carl bas asMuseaed ing. 3.8 (a) 37 REINFORCEMENT SPLICING ‘As far as possible the splices provided shall be away from te section of maximum suess and be staggered. ‘Aspe 1S:456-1978 recommendations, the sles nthe exural members should note at he sections where bending moment is more than $0 percent of he momen of resistance, and not rore than hal de bars shall be spliced ata section, When this is not avoidable, special precautions shall be taken suc as increasing the length of te lp andr sing a spiral around the length of the spice. The diferent types of spices ae as follows Lap splices: The lap length including anchorage valve of hocks in exua tension hallbe Lor 304, whichever is reac, and for direct tension 2 L, or 30, whichever i greaicr. The straight length ofthe lap shall ot be 4ess than 15 ¢ 0200 mm, The lap iength in compression shall be equal © L, oF 24 whichever is great. When bas of two different diameters ret be spliced, te lap length stall be calculated on the has ofthe iameter ofthe smaller bar “The eplces shall be considered staggered ifthe centre-to-centre distance of splices is more than the Lap length. Lap splices are not used for bars lage than 36 mun ¢ In such a case te bars may be welde Welded splices and mechanical connections: Te desiga strength of te welled spice oc mechanical connection sal be taken as equal w 80 percent of the desi suengih of he bar for tension splices, and 100 per cxnt of the design strength for cpression splices, However, 100 percent of the design suength may be assumed in tension when the spliced area forms not more than 20 pr Cent ofthe ttl area of steel atthe section and the splices are staggered atleast 600 mun from each ober: ‘The following examples will ilustate he procedure andthe types of problems encountered in practice. ‘Example 33: A simply suppones senforcd concrete rectal beat of size 250 500 mun effective is reinforced with six bars of 20 men @ at the ott and two brs of 16 m4 tthe 109 ofthe beam, The team caries unifonnly distributed sevice lad of 55 KN inclusive of self-weigh) over an effecuve span | | 1 ‘Shear, Bond and Torsion 77 CL. Bars b Bars a C.L. mid-span [+ Theoretical cutoff location for bars b ‘Total eabedrent (a) Simply supported beams Point of inflection Theoretical cutoff point for bars b Continuous bean (positive steel) Bars # Point of inflection CL. (b) Continucus bean (negative steel) lg 38 Certain pla for enforcement as simply evpportad beam i conan eam Rep-s 16 Reinforced Concrete Design of 6 m. The beam resis on 300 mm wide suppor. If he materials used are MIS grade concrete and HYSD Stel of grade F415, determine the development lengths required forthe bars on tension and compression ‘Sides, from the face f the suppon and also at midpan, Solution. For MIS grade coverete and Fe grade scl: Ga = 5 Nin, 0,» 230 Nina’, m= 19 snd j= 09 = 06 x(1 + 04) = 0.84 Nin For the given section, b= 250 mm, d= $00 mm: ‘Ag(6X20 im §) = 1885 man? Aq(2x 16mm §)=201 mu, Reaction atte support, V = wl = 55 62 = 165 EN. ‘Bending moment at the face ofthe support: ‘My = (165 x0:302) — (55 «0.30 x0.3072) «22.275 kN, {@ Development length onthe tension side: At suppon: gm MA i) = (22.275 x 1188S 0.9 x 50) = 2626 Nin” Ly= 90, MA 9) = 20 262544 x 0.84) = 156.31 mm, (i) Development length on the compression side: ‘Assume the section wo be balancod = SOOM + 230/19 5)}= 1462 mm, ‘Assume covert the op stel as 40 mm; thea the sess in concrete athe level of the compression sce is riven by. y Ca/26.26/19) w [146.2 ~ 4094500 ~ 1462} siving, Ofq.= 0415 Nira? and oy = 1.5 mx oq,= 11.83 Nim, For MIS concrete wit HYSD stel basin compression, y= 1.05 Nina Ly 90, (4%) = 16 11. 8M4 x 1.05) = 45.1 mm, ‘To compute the development ength on citer side ofthe point of maximum moment, i, a the midspan, sssume thatthe tension tel is stressed tits design vale, Le, 230 Nim’: y= 2023044 x0.84) = 1369 mm, ‘Toe midspn being the point of maximum stress, the bars call be anchored on both sides ofthis point by 1369 ‘mm. The length available on cher side s 3000 ~ 50 (= end cover) = 2950 mm, and ene adequate. ‘Bxninple 34 A cantilever slab projecting 1.6 m beyond the face ofthe beam of ros section of sine 250 x 500 uum effective cartes a uniformly igeibuted superimposed service lad of 33875 kNin* ‘The slab bas a thickness of 125 mm with 10 man @ mild size bars at 100 mum cleat a clear cover of 15 mun, Determine the development length equi for ste fom the face of the soppot. The concrete used is of ‘ade MIS. The unit weigh of concrete is 25 &Nn* ‘Solution. For MIS grade concrete and mild steel reinforcement: a= S Nina, = 140 Now, j= 087 and ty = 06 Nin, Consider a tm wide strip of cantilever slab: selfoweight w= 0125 x 125 ‘superimposed load, {otal Joa, w +6, Bending moment at he face of he support Moa wh = 7 (1.6972 = 896KNan | i Shear, Bond and Torsion 79 Stee in tesion consis of 10 mm § @100 mm cs, ie: ‘Ae (1x 14) x 1001100 785.4 ne wih Etestive deh d = 125 (2) ~ cer cover = 125 $— 15 = 105m, Assuming balanced section, sss in sel athe soppon: 5 = MiAg i) = (8.96% 1OEW7RS A XO87 x 105) » 124.88 Ninn? Development eng = of, HA) = 10% 124.884 x0.6)= 520.3 eum (ay $25 ma) Face ofthe support being the point of maximum sess, he tr hal be anced Bou ses ofthis yoin by $25 mm, Forte canilever prio, te leap aval 1600 ~ 50 1530 mm (> 525m) and Bence adeqate. For connection wih be Beam th anchorage provided is shown in Fig. 3.9. : 3.875 k/n 10mm @ 100m o/c 250 am Fig 39 Anchornge fr caterer da relaforcement Example 3.5: A reinfroed conrete team of rectangular cress seton of size 300% 600 men effective depths reinforced with five bars of 29 mum $0 te tension side. Te beam cares a uniformly distbutod tend of 12 KNin over an effective span of 8m. From te considerations of flexure anchorage and boo, ‘Seerminethe distance onthe span wer () 2 busca be cara ad i) Une trs can be Be UP ‘The materials sed are M15 grade crcre nd mld el renfrcemen. ‘Solution. For MIS grade concrete and mad sel reinforcement 140 Nin f= 250 Nin j= 0869, = 06 Nim? Fer the give section (b= 300 mum, d= 600mm, Ay (520 mm ¢)= 1570 ma? ‘Maximum bending moment, M = 12817096 kN, Moment of resistance wit respect oesion set . 1M, = 140 x 1570 x 0869 x 600) = 114.6 kN > M, ‘Anchorage length fo the 20 mm bar (assume 6,= 04 0): y= $66 a) = 20x 1404 0.6) = 1166.67 mm. (9 Two bers are tate curled. Moment of resistance wih respect to continuous hars (3 nos.) 180 Rebyjored Coneree Design My = GIS) x 1146 = 68.76 Ne. ‘The tcoretical cutofT pont i the loraion where bending momen reduces Lo this vale. Let It be at 2 ‘sence Y trom the centre, then PIAS - 6876) = 10146) siving y = 253m > L,(= 1.1660), “The bars mast satisfy the following contons: () Atte bars must comtine towards the suppor by L psd 12 (Le, 600mm or 240 mm, whichever 's greater) = 1766.67 mm from the cen ofthe beam at Which the peak sess oocar (The curated bars must continue beyond the heoresical point of uot" by at east doF 12.4, be. 600 ‘mum or 240 mm. Ths te curtailed bars have tobe continued up to distance of 3.13 m from the cea. ‘The second condition controls the curuilment distance of bar. Adehuae anchoage length of continuing ‘bars beyond the theoretical point of cut has toe provided. Length provided = 4000~ 2530 (Le, 1470 mm) + extension of bars inside supports + effect of hooks >, Hence providd length is moch more than required ‘tthe supports, Vi 12% 8/2 = 48 KN, Consider Ly 124, 2,240 man, then ‘MV = 166.67 ~240 (= 92667 man). ‘Actual MV = (68.26 x 10/48 x 109 = 14525 mon > 926.67 man. ‘ence reinfrcement is satisfactory and two bars canbe curtailed at 3130 mm fromthe cee. ‘Bending moment atthe cto point, i, 870 mam (= 4000 — 3130) frm the support: ‘M = 48 x087- 12 (087/72 = 37.22 Nim> 1768.16 4Nm), ‘Therefore addtional shear reinforcement aea of not ess than 04 BS, ito be provided over a distance (00.75 d trom the cutoff poi. ‘Shear force atthe eutoff point = 48 ~ 120.87 = 37.56 KN, 100 Aa lbd = 100° 942/(300 x 600) = 0.523, 520295 Nim’ [Nominal shear ses, y= (37.56 10°1(900 x 600) = 0.209 Nim’ Since 1, 7. then oth transverse and logit! reinforcements will be required fr resisting equivalent shear free and equivalent moment, respectively Longitudinal Steet Ti + DDVI.T = 30 (1 + 490B00)1.7 = 46471 EN n> M(=35 KN). ‘Since M, exceeds the numerical value of M, the longitadinal tel shall be provided on the flexural ‘compression face to resist the eqavaleat moment, Mg (= Mf, M) in addon to steel aquired for equivalent ‘ena moment, My (= M+ Bf: My 235 46471 = 81471 Nm, ‘The design ofthe section for flexure may result ina singly reinforced balanced, under-renforced or doubly reinforced section. To ascerai this contd he section tobe balanced and Mua Q bf = 0.897 x 300x (650)?= 54.49 Nn Since My > Mg itis a doubly reinforced section Tension reinforcements are: ‘Aas ® Ms jd) = (54.49 x 10°V230 x 0.906 450) = 581 ma? Ana™ Ma Mu My (d ~)} =(BLATI ~ $4.49) > 101230 x 450 ~ 40)] = 286 mam? ence, A Aq * Ag = 581 +286 = 867 mm? Depth ofthe erica! neutral axis, x,= kd = 0.283 «450 = 127.35 me, Compression see: Aus = My Mu M(VS m~1) Oa (= (4-5 = (BLATI ~ 5449) > 1OM(.S x13 ~ 1) 7 x (127.35 — 403450 - 40V/127.35} Since > M, the additional are of tel Aya atthe compression face ca be detemined a Aua= OM ~ HMMS m= 1) Or (4, Kd V8) Le OM, = Ma) = 315 ma ‘oval compression steel, Ay = Aus + Ara 1056 mm, Provide 3harsof22 um @ (A, = 1140 ma?> 1056 mar’) on compression face and3 bars of 20 ma § (Ay += 942 mun! > 867 mm on te tension side. ‘Since d > 450 mm, adonal steel of 0.1 per cent of the web areas to be provided equally along the vencal cer ‘A, 0.108100 = 0.10 300 x 450/100 = 135 mm? Provide 210 mum 6 (Ay = 157 min*> 135 mn’), one along cach vertical fae ofthe beam, Spacing of bats (4502) = 225 mun < 300 mem (te wid. Transverse Reinforcement Percentage of tension reinforcement, = 100 Adal = 100 982/(300 x 450) = 0.698, P= 0.698 pe cen shear strengt,t, = 0338 Ninn (fom Table 3.1) he for of vertical sirupsis required Consider 8 min ¢2-egged Fests Since > t transverse ste ‘tee timups (Ay = 100 mu). Then $.= Ay oe dT) + V2.9) fe dstnce between comer bars along width = 300 25 ~ 25 - 20 = 230 um and fe dstane between conier bars i We dgetion of depth = 450 - 25-11 = 414mm. Thus, 8, = (100 280% 4151030» 107230) + 90% 1072.59] = 66:85 man, where 6, ‘Shear, Bond and Torsion 87 ‘The spacing of vertical stirps stall be the eat ofthe folloving: Ax oof ~ 698] = 10023011407 ~ 0.338) x 300) = 71.72 mm, Gi), (= the shoner dimension of sir) = 300-25 ~ 25 +8 = 258 mm, Git) G+ yl = 58-4 442) = 175 mm, Gv) 075 4=0.75 x450= 337.5 mm, or 4 © mm, ‘Thos therequized spacing of mm §2-legged Fe415 grade tel vertical stirupsis 66 mm cenre-to-centre. However, this spacing of 66 mm of vertical imupsis too smal to facile proper compaction. Therefor, ‘normaly, minioum spacing i ited to 100 mm, Consider 10mm ¢2-legged stirups (A, = 157 mi) ‘The revised spacing can be obtained a: ior 10 sian) Sf & mg Degg sips Eg as Qiu) Henze, provide 10 mn @ 2egged verucal srups 100 mm ci. The reinforcement deals ae given in Fig. 3.18, 4 | | 25(clear cover to f Lesteel) T t - -22 @ i we aaceeet 2-10 o 4D yan i; me T Ky = 258 Fig 314 Relaforcemeat details forthe section subjected to torsion 39 PROBLEMS + Prob, 3.1: (@) Explain the terms shear stress, diagonal (ension, bond stress and development length with ‘ference wo reinforced concrete eas. (@) Discus the uty of bent up brs in reinforced concrete beams in resisting the shear, () Derive the expressions for computution of bond stress and shear stress in case of reinforced concrete ‘beams of rectangular cross-section with tension reinforcement of diameter 4. Also oblan the eatenship| between shear suess and bond sues, ‘Prob. 3.2 b reinforced concrete beam of rss section of size 300 550mm elective cris auaiformly Aistibued service load of 40 kN inclusive ofself-weight over a span of Sm, Ifthe tension einfercement ‘consists of 6 bas of 20 mm 9, determine he shear reinforcement. The materials wed are MIS grade concrete ‘and mild steel reinforcement Uo (Ans, Provide 8 mm § 2-leged sips @ 214.16 mm ce) Prob. 3.3: A reinforced concrete cantilever beam of rectangular cross-section of size 300 x 700 mm ‘vera carries a uniformly distributed service load of 30 kN inclusive of is sel-weight overa span of mn ‘Tae tension reinforcement consists of 6 mild steel bar of 20 mm 6, I concrete weds of grade MIS; detemnine sear reinforcement and check for anchorage. (Ans. d= 665 mm; provide 8 mm 6 2-Legged straps @ 214,16 mm ck; anchorage ie adequate) Prob. 34: A sioply suppored renfurced concrete beam of rectangular cross-section ofsze 300 x 700 ‘mm overall caries aconceniated loaf 125ENat | m from one suppor, inadddon to aunifermly disuibated Jad of 20 kN over a clear span of Gan, Th ston is enforced wis si 20 mm bare oC HYSD atest of srade Fo41S. The concrete used is of grade MIS, Design shear reinforcement when (only vertical sirups are provided, i) wo of te six bars ae beat up at 45" athe sume cross-section, and sig ft baat bent up 45" into ros cach consisting of as. The two groups ae spre by $00 mun (Ans, Wid d= 700 ~25 (clear cover) ~ 10 = 665 mm, ane 8 mun ¢2-egged sts () S, = 167-74 me, Gi) S,=395.48 mm, and Gi) $= 335.48 mn] Prob. 3.5: 125 mm hick simply supported sab, enforced with 12 mum @ bars at 100 mum cf, carries ‘sumifonnly distributed service load of 10 EN inclusive fit sef-weight. The mavals used are MIS grade ‘concrete and FeAl grade sce, Examine sear ses, i We span of te slab is 4m and clear cover of 13 mn, (Ans. t= 0.1923 Nan 1, = 0.3808 Nima’: shear reinforcements wot rquired) Prob. 36: A inforced concrete cantilever bean projects 1.2 from te face ofthe suppor witha bearing ‘on the suppon of 1.75 m. The tension reinforcement consists of 3 bars of 20 mm ¢ provided srnght on the {op fae of the beam. Check the adequacy of the anchorage. I inadequate, suggest the maximo sie of the bar that can be used as reinforcement, Te materials used ae MIS grade concrete and Fo41S grade steel, The Side cover is 50 mam, (Ans. In cantilever portion the anchorage provided is wt sulicient use 5 bas of 16mm 4) rob. 37: A reinforced concrete beam of rectangular crost-ectlon of size 300 x 600 mum effective is ‘Provide wit 6 bars of 20 mm 9 on is tension face. Two of the bars onthe tension face are bent pat the ‘Suppor for shea. Determine the development leg required for bars fom the feof the are coluina of size 300 300 mm in tension apd compresion zones. The materials used are M15 grade concrete and mild sce of grate I (6, = 190 Nau, (Ans, Lyin tension = 1583.3 mam in compression» 1266.7 em) Prob, 38: A.120:mun tick reinforced concrete canlever tab projects 1.10 m beyond te face of intel ‘f ross-serton 225 x 200 mm The slab is to carry a uniformly dsited service load ot 7 ENA I it reinforced with 10 mm @ bars @ 280 mm cc. Determive the engl equited from the face of line. The materials used are MIS grade concrete and HYSD sec of grade FeSOD (assume og 275 Nise), (Ans Ly =2029 mn). Prob. 3: simply supported reinforced concrete beam of ler spanof 7S m has across section of size 200 x 500 mim effective depth. eis reinforced with S bars of 1G mm on Ue tension side. The beam cares uniformly disributed service ead of 20 Nin including its own weigh Determine: () th length ove which vorical straps are tobe designed and hee spacing and (spacing Of traps wn 3 ars are Bot wpa he ‘The materials used are MIS grade concrete and mild sie enforcement. Use 6 min 2egged sixops 1 shear reinforcement. (Aas. (Deng from suppons = 1.9m, S,= 104 man ee, and i S, at support = 208 mn ce) rob, 3.10: A reinforced concrete beam of crss-Secton of size 30 x $50 mm overall is provided with ight 16mm 9 ars 2s tension reinforcement in two layers at an elective cover of 80 mm. If te beam cats ‘a unifornly distributed service lad of 15 Nha over a clear pan of 8m, determine the nutes of bars to be continued to the support from bond considerations, The materials sed are MIS grade concrete and inl tel reinforcement, Take j= 09 and = 300mm, (Ans. Tare bars ae required atte support) Prob, 3.11: & reinforced conerete canopy slab projecting 1.8 m from a300 umm wide canopy bean has thickness of 100 mm at the fee end and 200 mm atts jncton with he canopy bean. The tb cares ‘uniformly dstribsted superimposed load of 1.0 kNAn”. The span of canopy beam Which may be assutned fixed athe soon is Sm. Design the canopy beam ifthe stipulated materials are MIS grade cooeres and ‘mild ses enforcement. The unit weight of concrete is 25 ENim- ‘ns, With overall depth of $00 mam (~ pae/10) d= 465 tum; Mf = 2.425 EN, T=20,1864Nan and V= 30.75 KN. (75 cn’: he minimum transvere sel is given by A, 2 0.85 bf: 12mm § 2-egged smups @ 133 mm oe will be sulicest} Prob, 3.12: Design reinforced concrete beam ofretangul cross-section of size 300 700 mm effective, subjected wo a bending moment of 95.0 KNn, shea force of35 EN and a tarsion of 30 KEN, The materials ‘ied are M20 grade concrete and mild sie enforcement (Ans, Ay 1845 mm, A, two 12mm hanger bas, 12 mm ¢2-leeged stinups 141 mn ck; 2 bas of 10 ‘mm ¢ on ach vertical face) Fro, 3.13: A reinforced concrete beam of rectangular cos section of tice 200 x 400 mam overall is subjected fo a moment of 26 KN, a shea Srce of 6D KN and arson of 25 LN. Design he secon if ‘he maerials used are M20 grade concrete and mild ste enforcement. The covert longitu sees 40 Aco 1027 ss Aca (Ans. A, = 656 mn; A, = two 12m 9 hanger bas: 10mm $2-legged vertical stiups 130 mun cle) 1 Read mca | Sh oko 8 { | 4 DESIGN OF SLABS 4LINTRODUCTION Jorcinferod concrete construction, tesla is very widely vied seca eemetfomming ors and roots ‘ofthe buildings. A coneee slab isthe plane element having the depth D moch salle hn ts span and width may be supported by enforced cone: beams, by masonry wall or direc by clus rasta caries niformly ditibuted gravity loads acting normal ts ste and teste same tothe sports by ‘eur, shear and gon Slabs are classed according oe sym of suppor a: () one-way abe Fig 41a), (2) two-way labs (Fig. 4.10, i) at slabs supported direc en columns witout beams Fig. #1), (eee and now-ectangulr slabs, and (grid or walle slabs, ‘Slabs supported on two paral sides carry load by xu inthe dection pespendiclr othe Suppo Such lab ar called one-way slab and exenall ats sallow trams wits ge wid ad have Been ) ‘Two-way slab supported on beans (©) Slab supported directly on columns Fle 4.1 Diteent pet fsb stone 92 Retyjorced Concrete Design Ina continuous one-way slab where itis idealized by a scres of continuous long strips, as usted in Fig. 4.1 (@, te moments and shear foroes may be cbualned by using the coefficients given in Tables. 4.1 and 42: In this approach, redistribution of moment is not permed. The moments may als be determined by ‘moment distibuion or by anyother conventional method. In tke lau spproach moments ithe supports may be reduced by 15 per cent and thse atthe centre are incréase by 15 per cont. ‘Table 41 Beading Moment Coefcents Span manent Seppor omens Neate midil | Atthemiatloat| Atte oppor] At other rior aetna Witate |Gotanarpa | curoatas | “Spee Teaiaaaiapetei van | 7 —| — ino] Inge et im sane] ‘Nowe: For obtaining beading momenta concent al bemultpid by he tl erga ad ud the elec pan ‘Aitbe ead op ‘Atti oppor at nd pT ‘Atal teint ‘Type of load pt Port ‘or enppert ‘Ouerside [loner side Deatioadwndinposadiad(fnediw,| 04 os 05s os Taped ad (ot ad oes ts 06 0 ‘Now: Fr obtining the scar force, te coefficient sal be multiplied bythe tl Sexiga fond. Inthe 2ones of negative momenisat he inlrior suppor, the lad onthe slab scarred by lexare i both transverse and longitudinal diectons, esulng in reduced raoment in a transverse direction to at obarined ‘by one-way sab action. Adequate reinforcement sw be provided for lngitadinal moment near the inerich sippons, ‘The following are the design consideration fr one-way slabs (1) For a simply supported slab, the effective span is the clear span plus the effective depth, of centre tocente distance of suppons, whichever isles. (@) Tee main reinforcement, 1, the enforcement calculated for design moment, is placed in the Ces. Jayernear othe extreme fice ata stpulsed clear cover wo obtain he maximum effective depth, The secondary reinforcement oresiat, and shrinkage sess is provided perpendicular tothe main reiafercement sts tp. These barsalso keep tbe main reinforcement inposion and dstbutetheconcentatedor noe anor load throughout the tab; thus they may also be ealled distribution siel. The secondary reinforcement is at 4east0.15 percent fr mild steel, and 0.12 per ceat for HYSD stes of cross-sectional area of concrete. The ‘main relaforceneativ nocae shall be es than this amount. The diameter of enforcing Bars shallot excoed ‘one-ighth ofthe total thickness ofthe sla, Ina continuous sab atthe support top reinforcement is provided as main steel to resis egatve beading moment. In this ease, disubution Stee is placed under te maa tel, “The spacing ofthe reinforcing bars ina slab is given by ‘S= (area of one bar x 1000)/required area pe metre widh of sla in mum") ‘The spacing of main enforcing bars shall not be more an three times the effective dep ofthe slab (Gab or 450 mm, whichever is smaller. The spacing of secondary reaforcement bars shall pot he more than five times the eflecive dpa of De slab (Sd) or 430 mum, whichever i smaller. The minimum clear eover ‘reinforcement ina slab shal not be less than the diameter of such ab. 0 OQ OG OO OQ) OO EE EEE Vee ee Design cf Slate 93 ‘Innormal eases the shear srength of concrete in labs is greater than the nominal shear stress and shear ‘einforcement isnot required. 1S:456-1978 recommends an fnrease inthe sear srengh of concrete i slid ‘abs thier than 300 mm by a factor k given in Table 3.2. I shea is critical in a slab, the depth ofthe sib is increased. For checking, the development lengh L, may be assumed as 84 for HYSD bars for which end anchorage {snot usually provided and 12 ¢ for mild tel wih hooks, Design Procedure ‘The steps involved in the design ofa one-way sab are: (1) Forte stipulated materials determine te design parameters fr balanced section, (@ Estimate the effective span ofthe sab asthe east of the following (@) cleat span + effective depth ofthe slab (or width of support, whichever is less) (©) cenre-to

Das könnte Ihnen auch gefallen